Akut leukémia - leírás, tünetek (jelek), diagnózis. Akut mieloid leukémia (akut myeloid leukémia) Akut limfoblasztos leukémia kezelése

A krónikus limfocitás leukémia egy onkológiai betegség, amelyben atipikus B-limfociták halmozódnak fel a májban, csontvelő, nyirokcsomók, lép és perifériás vér. Fejlődésének kezdetén limfocitózisként nyilvánul meg, fokozatosan válik további tünetek. A csökkent immunitás miatt a betegek gyakran szenvednek különféle fertőzésektől. Fokozott vérzés és petechiális vérzés is előfordul.

A legtöbb szakértő egyetért abban, hogy a limfocitás leukémia kialakulásának fő oka az örökletes hajlam

A krónikus limfocitás leukémia progresszióját a következő változások kísérik nyirokcsomók:

  1. A nyak és a hónalj területén a nyirokcsomók mérete megnő.
  2. A mediastinalis csomópontok és hasi üreg.
  3. Az ágyéki terület nyirokcsomói az utolsó érintettek. Tapintással megfigyelhető mobilitásuk és nagy sűrűségük.

A krónikus limfocitás leukémia kialakulásával a csomópontok mérete elérheti az 5-7 centimétert, ami észrevehető kozmetikai hibát jelent.

A máj, a lép és a nyirokcsomók erős megnagyobbodása miatt a közeli szervek összenyomódásnak vannak kitéve, ami funkcionális rendellenességek kialakulásához vezet munkájuk során.

A krónikus limfocitás leukémia fő tünetei:

  • krónikus fáradtság;
  • súlyos fáradtság;
  • a munkaképesség általános csökkenése;
  • alvászavar;
  • szédülés.

A látens formában jelentkező betegség vérvizsgálat során véletlenül kimutatható. A betegség kialakulását általában a limfociták növekedése jelzi (akár 85-90%). Ugyanakkor a vörösvértestek és a vérlemezkék szintje normális. Ritka esetekben a betegeknél thrombocytopenia alakulhat ki.

A betegség előrehaladott formáiban a következő klinikai megnyilvánulások lehetségesek:

  • fokozott izzadás éjszaka;
  • fogyás;
  • enyhe hőmérséklet-emelkedés.

Általában a betegek immunfeszültségének csökkenését tapasztalják, ami urethritis, cystitis, valamint vírusos és bakteriális jellegű betegségek gyakori kialakulásához vezet. Még egy kis seb is beborulhat, és gyakran fekélyek jelennek meg a zsírszövetben.

Fontos! Pontosan fertőző betegségek limfocitás leukémiában halálhoz vezethet.

Elég gyakran tüdőgyulladás alakul ki a betegség hátterében, ami a tüdő szellőzésének megsértését okozza a tüdőszövet összeomlása miatt. A krónikus limfoid leukémia másik szövődménye az exudatív mellhártyagyulladás, amely a mellkasban elhelyezkedő nyirokcsatorna összenyomódásához vagy szakadásához vezethet.

Nem ritka, hogy limfocita leukémiában szenvedő betegeknél generalizált herpes zoster alakul ki.

A ritkább szövődmények a következők:

  • halláskárosodás;
  • ideggyökerek, agyhártyák és velősejtek limfociták általi beszivárgása;
  • fülzúgás előfordulása.

A limfocitás leukémia Richter-szindrómává fajulhat, amelyet diffúz limfómának neveznek. Ez utóbbi esetben megfigyelhető gyors növekedés nyirokcsomók terjedéssel kóros folyamat nem kapcsolódó szerveknek nyirokrendszer. A betegek mindössze 5-6%-a éli túl a limfocitás leukémia e szakaszát. A halál a fejlődés eredményeként következik be belső vérzés, vérszegénység, a beteg kimerültsége és fertőzések okozta szövődmények. Akut kialakulása is lehetséges veseelégtelenség a veseszövet limfociták általi beszűrődése miatt.

Diagnosztika


Krónikus limfocitás leukémia esetén vérvizsgálatot kell végezni a vérben lévő limfociták számának meghatározására, amelynek növekedése a betegség kialakulásához vezet.

Az esetek túlnyomó többségében a betegséget teljesen véletlenül fedezik fel. Általában ez más betegségek vizsgálatakor történik. Egyes esetekben a limfocitás leukémiát rutinvizsgálat során fedezik fel.

A pontos diagnózis felállítása egy átfogó diagnózis eredményeként lehetséges, amely a következő lépéseket tartalmazza:

  • a betegek panaszainak meghallgatása és anamnézis felvétele;
  • ellenőrzés;
  • a vérkép megfejtése krónikus limfocitás leukémiában és az immunfenotipizálási eljárás eredményei.

A krónikus limfocitás leukémia diagnosztizálásának fő kritériuma a limfociták számának növekedése a vérben. A szakértők a limfociták immunfenotípusát is vizsgálják.

A laboratóriumi diagnosztika során a következő normától való eltérések azonosíthatók:

  • a limfociták mérete megnövekedett;
  • vannak Gumprecht árnyékai;
  • kisméretű B-limfociták vannak jelen;
  • Atípusos limfociták észlelhetők.

A betegség stádiumát a nyirokcsomók vizsgálatával határozzák meg. A kezelési terv elkészítéséhez citogenetikai diagnosztika szükséges. Ha a szakember limfóma kialakulását gyanítja, a pácienst biopsziára utalják. Lehetőség van szegycsontból (szegycsontból) csontvelő punkció elvégzésére.

Osztályozás

Figyelembe véve morfológiai jellemzőkés a betegség tüneteit, valamint fejlődési ütemét és a terápiára adott válaszát, ennek a betegségnek számos formája létezik:

  1. Krónikus limfocitás leukémia, amelyet jóindulatú lefolyás jellemez. Ezzel a betegséggel a beteg hosszú ideig kielégítő állapotban marad. A leukociták száma a vérben lassan növekszik. A diagnózis után a nyirokcsomók évtizedekig normálisak maradhatnak, a betegek normál életmódot folytatnak, megőrizve munkaképességüket.
  2. Tumor forma. Jellemzője a nyirokcsomók kifejezett megnagyobbodása az enyhe leukocitózis hátterében.
  3. Klasszikus forma. Progresszívnek is nevezik. A lassan fejlődő jóindulatú formától eltérően ennek a patológiának a tünetei több hónapon keresztül fokozódnak. A beteg állapotának romlásával párhuzamosan a nyirokcsomók megnagyobbodása következik be.
  4. Csontvelő forma. Progresszív cytopenia jellemzi. A nyirokcsomók, a lép és a máj nem növekszik meg az ilyen típusú krónikus limfocitás leukémiában.
  5. T sejt forma. Nagyon ritka betegség, amely csak az esetek 5%-ában alakul ki. A dermis infiltrációja kíséri, és gyors progresszió jellemzi.
  6. Szőrös sejtes leukémia. A nyirokcsomók nem megnagyobbodnak, de lépmegnagyobbodás és cytopenia észlelhető. A sejtszerkezet mikroszkóp alatti vizsgálata azt mutatja, hogy a limfociták citoplazmájában jellegzetes törések, valamint a széleken bolyhos hajtások jelennek meg.
  7. Limfocita leukémia megnagyobbodott léptel. A patológia legszembetűnőbb tünete a szerv méretének növekedése.
  8. Prolimfocita forma. A csontvelőből és a vérből, a nyirokcsomókból és a lépből vett limfociták sejtmagokat (nukleolusokat) tartalmaznak, amelyeket az érett sejtek általában nem tartalmaznak.
  9. Limfocita leukémia paraproteinémiával. A tünetek hasonlóak a fent leírt betegségekhez. További tünet a monoklonális G- vagy M-gammopathia.

A megnyilvánulás mértékétől függően a betegség három szakaszát különböztetjük meg:

  1. A kezdeti. Nincsenek klinikai tünetei, és véletlenszerű diagnózis során észlelik.
  2. Kifejlett klinikai megnyilvánulások.
  3. Terminál. A betegség előrehaladott stádiumban van, és gyakran halálhoz vezet.

A kezelés jellemzői


A dózist és a kezelési rendet egyénileg határozzák meg, a beteg állapotától függően.

A krónikus limfoid leukémia kezelését a betegség stádiumától és a beteg egészségi állapotától függően végzik. Például, ha a betegség be van kapcsolva kezdeti szakaszbanés nincs semmilyen megnyilvánulása, akkor az orvosok kivárást választanak. Ez magában foglalja a háromhavonta végzett vizsgálatokat. A kezelést addig nem végezzük, amíg a betegség lefolyása súlyosbodni nem kezd. A patológia évtizedekig nem fejlődhet ki.

A terápia felírásának oka a leukociták számának legalább kétszeres növekedése rövid időn belül (legfeljebb 6 hónapig). Ebben az esetben a betegnek leggyakrabban kemoterápiát írnak fel, amely a következő gyógyszerek kombinációját foglalja magában:

  • fludarabin;
  • Rituximab;
  • Ciklofoszfamid.

Ha a krónikus limfocitás leukémia tovább fejlődik, a beteget felírják nagyszámú hormonális szerek és előkészületek kezdődnek a csontvelő-átültetési műtéthez.

Veszélyes idős korban a kemoterápia és a műtét elvégzése. Ezért az ilyen betegek monokemoterápiát (Chlorambucil) írnak fel, vagy kombinálják ezt a gyógyszert Rituximabbal.

Előrejelzés

Jelenleg egyetlen esetben sem fordul elő teljes gyógyulás a krónikus limfoid leukémiából, de lehetséges a hosszú távú remisszió.

A túlélési prognózis számos tényezőtől függ, beleértve a beteg korát, nemét, egészségi állapotát, a betegség időben történő diagnosztizálását stb. A túlélés mértéke általában nagyon széles tartományon belül változhat - néhány hónaptól több évtized.

A betegséget bizonyos kiszámíthatatlanság jellemzi. Egyes esetekben a kedvező prognózisú betegek meghaltak a betegség szövődményei miatt.

Információ: A LEUKEMIA olyan kifejezés, amely a vérképző rendszer számos daganatát egyesíti, amelyek vérképző sejtekből származnak és a csontvelőt érintik. A leukémia két fő csoportra - akut és krónikus - felosztását a daganatsejtek szerkezete határozza meg: az akut leukémiát foglalja magában a leukémia, amelynek sejtszubsztrátját blastok képviselik, és a krónikus leukémia, amelyben a tumorsejtek nagy része differenciálódik. és főleg kiforrott elemekből áll. A betegség időtartama nem határozza meg, hogy egy adott leukémia akutnak vagy krónikusnak minősül-e. Etiológia, patogenezis. Emberben az akut leukémia és a krónikus mieloid leukémia okai lehetnek a kromoszómális apparátus összetételében és szerkezetében fellépő, örökletesen meghatározott vagy bizonyos mutagén tényezők hatására szerzett rendellenességek. Az egyik az ionizáló sugárzás. A leukémia kialakulásának oka a kémiai mutagének hatása is. A benzolnak kitett személyek, valamint a citosztatikus immunszuppresszánsokat (imurán, ciklofoszfamid, leukarán, szarkolizin, mustargén stb.) kapó betegek körében az akut leukémia növekedése bizonyított; az akut leukémia gyakorisága ebben a betegcsoportban több százszorosára nő. Ismertek tények az akut myeloblastos leukémia, az akut erythromyelosis, a krónikus limfocitás leukémia hosszú távú kemoterápia hátterében, a Waldenström-féle makroglobulinémia, a myeloma multiplex, a lymphogranulomatosis és más daganatok előfordulásáról. Kimutatták az örökletes rendellenességek szerepét a mieloid és nyirokszövetekben, amelyek hajlamosak a leukémiára. Leírják a krónikus limfocitás leukémia domináns és recesszív öröklődésének megfigyeléseit; egyes etnikai csoportokban ennek a leukémiának alacsony, míg másokban megnövekedett előfordulási gyakorisága figyelhető meg. Ezekben az esetekben gyakrabban nem maga a leukémia öröklődik, hanem a fokozott variabilitás - kromoszóma-instabilitás, amely hajlamosítja a szülő myeloid vagy nyiroksejteket a leukémiás átalakulásra. A kromoszómaanalízis lehetővé tette annak megállapítását, hogy bármely leukémiában a daganatos leukémiás sejtek klónja, egy kezdetben mutáns sejt leszármazottja elterjed a szervezetben. A rosszindulatú sejtek genotípusának instabilitása leukémiában új klónok megjelenését okozza az eredeti tumorklónban, köztük a szervezet élete során, valamint a gyógyászati ​​termékek a leginkább autonóm klónokat „kiválasztják”. Ez a jelenség magyarázza a leukémia progresszióját és a citosztatikumok kontrollja alól való kikerülését. A leukémia akut. Morfológiai (főleg citokémiai) kritériumok szerint az akut leukémia következő főbb formáit különböztetjük meg: limfoblasztos, myeloblastos, promyelocyta, myelomonoblastos, monoblasztos, megakarioblasztos, eritromyelózisos, plazmablasztos, differenciálatlan, alacsony fokú akut leukémia. Minden akut leukémiát a vérszegénység okozta fokozódó „indokolatlan” gyengeség, rossz közérzet, néha légszomj, szédülés jellemez. A megnövekedett testhőmérséklet és mérgezés a nem limfoblasztos akut leukémia gyakori tünetei. A nyirokcsomók, a máj és a lép megnagyobbodása előrehaladott stádiumban nem minden akut leukémiában fordul elő, de a terminális stádiumban az akut leukémia formájától függetlenül kialakulhat. Nem ritka az elsősorban thrombocytopenia által okozott hemorrhagiás szindróma: nyálkahártya vérzése, petechiális kiütések a bőrön, különösen a lábakon. Leukémiás blast infiltrátumok jelenhetnek meg a tüdőben, a szívizomban és más szövetekben és szervekben. Az akut leukémia diagnózisa a vér és a csontvelő citológiai vizsgálatán alapul, amely a blastsejtek nagy százalékát tárja fel. A korai szakaszban általában hiányoznak a vérből, de a citopénia kifejezett. Emiatt citopénia esetén akár egy csíra esetében is csontvelő punkció szükséges, amely ambulánsan is elvégezhető. A csontvelőben minden akut leukémiában magas (tíz százalékos) blaszttartalom található, kivéve az akut, alacsony százalékos leukémiát, amelyben sok hónapig a vérben és a csontvelőben a blastsejtek százalékos aránya alacsonyabb lehet. mint 15-20, és a csontvelőben ebben a formában Általában a blasztok százalékos aránya kisebb, mint a vérben. Az akut leukémia formáját hisztokémiai módszerekkel határozzák meg. A felnőttkori akut leukémia leggyakoribb formája a mieloblasztos és a mielomonoblasztos leukémia. A betegség kezdetén ezeknél a formákban a máj és a lép általában normál méretű, a nyirokcsomók nem megnagyobbodtak, ugyanakkor nem ritka a mélygranulocitopénia, vérszegénység és thrombocytopenia sem. A mérgezés gyakran súlyos, és a testhőmérséklet emelkedik. Az erősejtek szerkezeti magjai finom kromatinhálózattal rendelkeznek, gyakran több kis sejtmag; a blastsejtek citoplazmája azurofil szemcséket vagy Auer testeket tartalmaz, amelyek pozitív reakciót adnak a peroxidázra és a lipidekre. A mielomonoblaszt leukémiában nemcsak ezeket az anyagokat mutatják ki a citoplazmában, hanem a monocitikus sorozat elemeire jellemző alfa-naftil-észterázt is; az alfa-naftil-észterázt a nátrium-fluorid gátolja. Az akut limfoblasztos leukémia gyakrabban fordul elő gyermekeknél. Általában kezdettől fogva lymphadenopathiával, megnagyobbodott lépvel és ossalgiával fordul elő. A vérben először csak mérsékelt normokróm anémia és leukopenia figyelhető meg, de a csontvelőben - teljes blastosis. A robbanósejteknek lekerekített magjuk van, finom kromatinhálózattal és 1-2 sejtmaggal, valamint szemcsés, keskeny citoplazmával. A CHIC-reakcióval glikogéncsomókat észlelnek a citoplazmában, amelyek nyaklánc formájában koncentrálódnak a sejtmag körül. Az akut promyepocytás leukémia meglehetősen ritka; Egészen a közelmúltig gyors áramlás jellemezte. Súlyos mérgezés, vérzés és DIC-szindróma okozta hypofibrinogenemia jellemzi. A nyirokcsomók, a máj és a lép általában nem növekszik meg. A hemogram vérszegénységet, súlyos thrombocytopeniát és nagy százalékban atípusos blasztokat mutat a csontvelőben. Erőcellák különféle méretekés a formák citoplazmája sűrűn van kitöltve egyes sejtekben nagy ibolyabarna szemcsékkel, amelyek a sejtmagon helyezkednek el, másokban kicsi, bőséges azurofil szemcsékkel; Az Auer testek gyakoriak. A gabona savas szulfatált mukopoliszacharidokat tartalmaz. Ezeknek a leukémiás sejteknek a magjai a vérben gyakran kétsoros alakúak, alakjukat még gyakrabban nehéz megkülönböztetni a citoplazma szemcséssége miatt. A beteg halálának közvetlen oka leggyakrabban agyvérzés. Az akut monoblasztikus leukémia viszonylag ritka. Ennek a formának a tipikus megjelenése alig tér el a mieloblasztos formától, de a mérgezés és a testhőmérséklet lázas szintre emelkedése kifejezettebb. Gyakori tünet az íny nyálkahártyájának hiperplázia a bennük lévő leukémiás proliferáció miatt. A vérben kezdetben a granulocita vonal viszonylag megőrződött, a blasztok mellett sok érett, többé-kevésbé hibás monocita található. Az erősejtek bab alakú szerkezeti maggal rendelkeznek, több maggal és szürkéskék citoplazmával, néha csekély azurofil szemcsézettséggel. Citokémiailag pozitív reakciót mutat a nátrium-fluorid által elnyomott alfa-naftil-észterázra, gyengén pozitív reakciót peroxidázra és lipidekre. E betegek vérszérumában és vizeletében magas a lizozim szintje. Az akut plazmablasztos leukémiát plazmablasztok és plazmaciták megjelenése jellemzi a csontvelőben és a vérben, celluláris atípia jellemzőivel; ezen kívül sok differenciálatlan robbanás található. Az akut leukémia ezen formájának jellegzetes citokémiai jellemzői nem ismertek; jellemzője a paraprotein kimutatása a szérumban. Az extramedulláris leukémiás gócok gyakran kifejeződnek - megnagyobbodott nyirokcsomók, máj, lép, leukémia a bőrben, herék. Az akut megakarioblasztos leukémia nagyon ritka. A csontvelőben és a vérben megakarioblasztok (blasztos, de hiperkromatikus sejtmaggal, keskeny citoplazmával fonalas kinövésekkel rendelkező sejtek), valamint differenciálatlan blasztok jelenléte jellemzi. Gyakran csúnya megakariociták és sejtmagjuk töredékei találhatók a vérben és a csontvelőben. Jellemző a trombocitózis (több mint 1000-lO (negyedik fokig) µl). Az akut erythromyelosis viszonylag ritka. A betegséget vörösvértest-hiperplázia jellemzi, súlyos hemolízis jelei nélkül. Klinikai tünetek: normo- vagy hyperchromic anaemia progressziója retikulocitózis nélkül (általában legfeljebb 2%), enyhe icterus az erythrokaryocyták lebomlása miatt, fokozódó leukopenia és thrombocytopenia. A csontvelőben a vörösvértestek mennyisége megnövekszik a többmagvú eritroblasztok és a differenciálatlan Power sejtek jelenlétével. Az akut leukémia más formáitól eltérően a vörös tumorsejtek gyakran differenciálódnak az oxifil normocita vagy eritrocita stádiumába. Az akut erythromyelosis gyakran akut myeloblastossá alakul át. A neuroleukémia az akut leukémia, ritkábban a krónikus mieloid leukémia egyik gyakori szövődménye. A neuroleukémia leukémiás elváltozás (infiltráció) idegrendszer. Ez a szövődmény különösen gyakori az akut limfoblasztos leukémiában szenvedő gyermekeknél, és ritkábban az akut leukémia egyéb formáiban. A neuroleukémia előfordulását a leukémiás sejtek agyi membránokba történő áttét okozza és gerincvelő vagy az agy anyagába (prognosztikailag ez a daganatnövekedés súlyosabb típusa). A neuroleukémia klinikai képe meningeális és hipertóniás szindrómákból áll. Mark kitartó fejfájás, ismételt hányás, levertség, ingerlékenység, porckorongduzzanat látóidegek, nystagmus, strabismus és a károsodás egyéb jelei agyidegek és meningealis jelek. A cerebrospinális folyadékban magas a blast citózis. Az agy-gerincvelői folyadékban a magas citózis és blastsejtek kimutatása a neuroleukémia korábbi jele, mint a leírt klinikai kép. Az intracerebrális áttétek esetén citózis nélküli agydaganat képe van. Kezelés. Akut leukémia esetén sürgős kórházi kezelés szükséges. Egyes esetekben pontos diagnózissal citosztatikus kezelés járóbeteg alapon lehetséges. Patogenetikai kezelést alkalmaznak a remisszió eléréséhez citosztatikumok kombinált adagolásával, az összes nyilvánvaló és feltételezett leukémiás góc megszüntetése érdekében, miközben lehetséges a hematopoiesis súlyos depressziója. Az akut leukémia remissziója olyan állapot, amelyben a vérlemezkék szintje 1 μl-ben 10-104 felett van, a leukociták száma 3000 μl felett van, a csontvelőben kevesebb, mint 5% blast és kevesebb, mint 30% limfoid sejt. , és nincs csontvelőn kívüli leukémiás proliferáció. Gyermekek akut limfoblasztos leukémiájában a teljes remisszió kötelező kritériuma a cerebrospinális folyadék normál összetétele. Akut limfoblasztos leukémiában szenvedő gyermekeknél a leghatékonyabb kombináció a vinkrisztin, amelyet 1,4 mg/m2 (legfeljebb 2 mg) dózisban írnak fel hetente egyszer intravénásan, és a prednizolont szájon át naponta 40 mg/m2 dózisban. Ezzel a terápiával a gyermekek körülbelül 95%-ánál 4-6 héten belül remisszió érhető el. Már a remisszió elérésének időszakában megkezdődik a neuroleukémia megelőzése: az akut limfoblasztos leukémia diagnózisának felállítását követő napon el kell végezni az első spinalpunctiót, és intralumbarálisan metotrexátot (ametopterint) kell beadni 12,5 mg/m2 dózisban. . A spinális punkciókat metotrexát adagolásával a jelzett dózisban 2 hetente meg kell ismételni a remisszió eléréséig. Közvetlenül a remisszió elérése után speciális megelőző kúrát végzünk, amely magában foglalja a fej besugárzását 2400 rad dózisú kétoldali mezőkből, az 1. és 2. nyakcsigolyára kiterjedően, de a szem, a száj és a teljes terület védelmével. az arckoponya, és egyidejűleg ötszörös (3 hetes besugárzás) metotrexát intralumbális adagolása azonos dózisban (12,5 mg/m2). Ha a lumbálpunkció során neuroleukémiát diagnosztizálnak, a fej megelőző besugárzását megszakítják, a neuroleukémiát két citosztatikus gyógyszer intralumbális beadásával kezelik: metotrexát 10 mg/m2 (maximum 10 mg) és citozar (kezdő dózis 5 mg/m2) fokozatosan 30 mg/m2-re emeljük). m2). Az akut limfoblasztos leukémia remissziójának időszakában gyermekeknél folyamatos citosztatikus terápiát végeznek három citosztatikummal - 6-merkaptopurin (50 mg/m2 naponta), ciklofoszfamid (200 mg/m21 hetente egyszer), metotrexát (20 mg). /m21 hetente egyszer); a kezelés 3,5-5 évig folytatódik. Akut limfoblasztos leukémiára felnőtteknél és gyermekeknél kedvezőtlen kezdeti indikációkkal (későn megkezdett és megszakított kezelés a programban való részvétel előtt, 10-12 év feletti életkor, kezdeti leukocitaszint több mint 20 000 1 μl-ben) a remisszió első hetében a program keretében, beleértve a vinkrisztint, prednizolont és rubomicint, a citosztatikus kombinációk egyikét írják fel: COAP, CHOP vagy POMP. A COAP kombináció ciklofoszfamidból és citozárból áll, amelyet a kúra 1. és 4. napjától intravénásan adnak be 50 mg/m2 mennyiségben naponta háromszor fecskendővel; vinkrisztin 1,4 mg/m2 IV dózisban az 1. napon, és prednizolon naponta 100 mg/m2 dózisban az 1. és 4. napon. A CHOP kombináció a kúra 1. napján intravénásan beadott ciklofoszfamidból, 750 mg/m2 dózisban, adriamicinből - 50 mg/m2 intravénásan az 1. napon, vinkrisztinből - 1,4 mg/m2 (maximum 2 mg) az 1. napon intravénásan. és prednizolon, naponta adva a kúra 1. és 5. napjától, napi 100 mg/m2 dózisban. A POMP kombinációt 5 napos kúrára tervezték, amely magában foglalja a 6-merkaptopurin (purinetol) napi 300-500 mg/m2 szájon át az 1. naptól az 5. napig, vinkrisztint - 1,4 mg/m2 IV az 1. napon, metotrexátot - Naponta 7,5 mg/m2 intravénásan az 1. naptól az 5. napig, és a prednizolont szájon át naponta 200 mg/m2 naponta. Az egyik ilyen kurzus a remisszió kezdetén történik, hogy megszilárdítsa azt. Ezután (a citopénia megjelenése után - a leukociták szintje 3000 sejtre emelkedik 1 mm3-re) megkezdődik a remisszió fenntartását célzó terápia; akut limfoblasztos leukémiában folyamatosan ugyanazzal a három gyógyszerrel (6-merkaptopurin, metotrexát és ciklofoszfamid) végezzük, mint 2-10 éves gyermekeknél, de e terápia helyett másfél havonta szájon át tablettában, ill. a ciklofoszfamidhoz hasonlóan, porban, felváltva hajtsa végre a tanfolyamot. COAP, CHOP vagy POMP (a fenntartó terápia teljes időtartamára, azaz 5 házi kedvencre, válasszon kettőt ebből a három kúra közül egy adott beteg számára). Életkortól függetlenül az akut limfoblasztos leukémiában szenvedő betegeket két citosztatikus gyógyszerrel akadályozzák meg a neurolémia kialakulásától: metotrexáttal (10 mg/m2, maximum 10 mg) és citozarral (5-ről 30 mg-ra növekvő adagokban - összesen 5 intralumbális injekció) vagy fejre. besugárzás (dózis 24 Gy 15 alkalom) és metotrexát intralumbarálisan 5 alkalommal adva egyidejűleg besugárzással 12,5 mg/m2 dózisban. Akut nem limfoblasztos leukémiában a remisszió eléréséhez használt fő gyógyszerek a citozar és a rubomicin (vagy adriamicin). „7 + Z” kombinációban írhatók fel: a citozart 7 napon keresztül folyamatosan adják be 200 mg/m2 napi dózisban vagy naponta kétszer 12 óránként 200 mg/m2 dózisban 2 órán keresztül intravénásan; A rubomicint intravénásan, fecskendővel 45 mg/m2 (60 év felettieknek 30 mg/m2) dózisban adják be a kúra 1., 2. és 3. napján. A citozárhoz és a rubomicinhez 6-merkaptopurin adható 12 óránként 50 mg/m2 dózisban, míg a citozar dózisa 100 mg/m2-re csökkenthető, 12 óránként adva. A Cytosart 8 napig, a 6-merkaptopurint a 3. naptól a 9. napig adják be. A remisszió elérésekor a rögzítő folyamat - konszolidáció - ugyanaz lehet, mint ami a remisszióhoz vezetett. A remisszió fenntartásához használja a citozar és a rubomicin ugyanazt a kombinációját ("7 + 3" kurzus), havonta 2,5 vagy 3 hetes időközönként, vagy 5 napos citozart szubkután, 100 mg/m2-ben minden 12. órás kombináció (a tanfolyam első napján) citosztatikumok egyikével, mint például ciklofoszfamid (750 mg/m2) vagy rubomicin (45 mg/m2) vagy vinkrisztin (1,4 mg/m2 az 1. napon) és prednizolon (40 mg/m2) m2) m2 az 1. naptól az 5. napig) vagy metotrexátot (30 mg/m2). A fenntartó kezelést 5 évig folytatják, mint az akut limfoblasztos leukémia esetében. Minden beteget neuroleukémia profilaxissal kezelnek. Első lumbálpunkció a metotrexát 12,5 mg/m2 (maximum 15 mg) adagolásával minden korcsoportban az akut leukémia minden formája esetén az akut leukémia diagnózisát követő első napokban végezzük. Felnőtteknél a neuroleukémia megelőzésének fő szakaszát a remisszió elérése után hajtják végre; akut limfoblasztos leukémiában szenvedő gyermekeknél, még a remisszió indukciós időszakában is, a metotrexátot 2 hetente újra kell adni 12,5 mg/m2 (maximum 15 mg) dózisban. Reakciók esetén a prednizolont intravénásan írják fel 120 mg-os adag beadása előtt. A leukémia krónikus. Leggyakoribbak a limfocitás leukémia, myeloid leukémia, myeloma multiplex, erythremia, ritkábban a krónikus subleukémiás myelosis (osteomyelosclerosis, myelofibrosis), krónikus monocitás leukémia, Waldenström-féle makroglobulinémia. Krónikus mieloid leukémiában a daganatos folyamat a csontvelő granulocita, vérlemezke és eritrocita vonalait egyaránt érinti. A daganat progenitora a mielopoézis prekurzorsejtje. A folyamat átterjedhet a májra, a lépre, és a terminális stádiumban bármely szövet érintett lehet. A krónikus myeloid leukémia klinikai lefolyása előrehaladott és terminális stádiumra oszlik. Az előrehaladott stádium kezdetén a betegnek nincsenek panaszai, a lép nem vagy kismértékben megnagyobbodik, a perifériás vér összetétele megváltozik. Ebben a szakaszban a diagnózis felállítható a neutrofil leukocitózis „motiválatlan” természetének elemzésével, a képlet mielocitákra és promyelocitákra való eltolásával, a csontvelőben a leukocita/eritrocita arány jelentősen megnövekedett és a vérben a „Philadelphia” kromoszóma kimutatásával. granulociták és csontvelősejtek. A csontvelő-trefinben általában már ebben az időszakban megfigyelhető a zsír szinte teljes kiszorítása a mieloid szövet által. Az előrehaladott szakasz átlagosan 4 évig tarthat. Megfelelő terápia mellett a betegek állapota kielégítő marad, munkaképes marad, normális életmódot folytat ambuláns megfigyeléssel és kezeléssel. A terminális stádiumban a krónikus myeloid leukémia lefolyása rosszindulatú tünetekkel jár: magas láz, gyorsan előrehaladó kimerültség, csontfájdalom, súlyos gyengeség, gyors lép-, máj- és időnként megnagyobbodott nyirokcsomók. Ezt a szakaszt a normál hematopoiesis elnyomásának jeleinek megjelenése és gyors növekedése jellemzi - vérszegénység, thrombocytopenia, amelyet hemorrhagiás szindróma, granulocitopénia, fertőzéssel komplikált, nyálkahártya-nekrózis. A krónikus myeloid leukémia terminális stádiumának legfontosabb hematológiai tünete a blastos krízis - a blastsejtek tartalmának növekedése a csontvelőben és a vérben (eleinte gyakrabban myeloblastok, majd differenciálatlan blasztok). Kariológiailag a terminális stádiumban az esetek több mint 80% -ában meghatározzák a rendellenes számú kromoszómát tartalmazó hematopoietikus sejtek aneuploid klónjainak megjelenését. A betegek várható élettartama ebben a szakaszban leggyakrabban nem haladja meg a 6-12 hónapot. A krónikus mieloid leukémia kezelését a diagnózis pillanatától kezdik meg. Előrehaladott stádiumban a myelosan 2-4 mg/nap dózisú terápia hatásos (legfeljebb 6 mg/napot írnak elő 100 000/1 mm3 leukocitaszint felett). A kezelést járóbeteg alapon végzik. Ha a myelosan hatástalan, myelobromolt írnak elő (jelentős lépmegnagyobbodás esetén lép besugárzása végezhető). Amikor a folyamat a terminális szakaszba lép, citosztatikus gyógyszerek kombinációit alkalmazzák, amelyeket általában az akut leukémia kezelésére használnak: vinkrisztin és prednizolon, VAMP, citozar és rubomicin. A terminális szakasz elején a mielobromol gyakran hatásos. A krónikus limfocitás leukémia az jóindulatú daganat immunkompetens rendszer; a daganat morfológiailag érett limfocitákon alapul. A betegség kezdete gyakran nem határozható meg: a beteg teljes egészségi állapota és a kellemetlen szubjektív érzések hiányában a vérben kicsi, de fokozatosan növekvő limfocitózist észlelnek. A korai szakaszban a fehérvérsejtszám normális lehet. Jellegzetes jel betegség - megnagyobbodott nyirokcsomók. Néha növekedésüket a vér változásaival egyidejűleg, néha később észlelik. A lép megnagyobbodása gyakori tünet; a máj kevésbé valószínű, hogy megnagyobbodik. A vérben a limfociták számának növekedésével, egyedi prolimfociták és néha ritka limfoblasztok jelenlétével együtt gyakran megfigyelhetők a krónikus limfocitás leukémiára jellemző úgynevezett Gumprecht-árnyékok - a kenet készítése során elpusztult limfocitamagok, amelyben a kromatin oldalai között nukleolusok láthatók. A betegség előrehaladott stádiumában a neutrofilek, a vérlemezkék és a vörösvértestek tartalma évekig normális szinten maradhat. Krónikus limfocitás leukémia esetén a limfociták nagy százaléka található a csontvelőben. A betegség kialakulását gyakran a gamma-globulinok általános szintjének csökkenése kíséri. A humorális immunitás elnyomása gyakori fertőző szövődményekben, különösen tüdőgyulladásban nyilvánul meg. Egy másik gyakori szövődmény a cytopenia, gyakrabban vérszegénység és thrombocytopenia. Ez a szövődmény összefüggésbe hozható az eritrociták és vérlemezkék, illetve az eritrokariociták és megakariociták elleni autoantitestek megjelenésével. De nem ez az egyetlen mechanizmusa a citopenia kialakulásának krónikus limfocitás leukémiában; a limfociták (különösen a T-limfociták) lehetséges elnyomó hatása az erythropoiesis vagy a thrombocytopoiesis prekurzor sejtjeire. A krónikus limfocitás leukémia terminális stádiuma, amely szarkóma növekedéssel vagy blastos krízissel nyilvánul meg, ritkán figyelhető meg, a blastos krízis különösen ritka. A lymphosarcoma kialakulását egyes esetekben a vér limfocitózisának neutrofiliává történő változása kísérheti. A szőrös sejtes leukémia a krónikus limfocitás leukémia egy speciális formája, amelyben a limfociták homogén, blastos magra emlékeztető sejtmaggal és a citoplazmabolyhos kinövésekkel rendelkeznek. E sejtek citoplazmája sok savas foszfatázt tartalmaz, amely ellenáll a borkősav hatásának. Klinikai kép lép megnagyobbodása, a perifériás nyirokcsomók enyhe növekedése és súlyos citopénia jellemzi. A lépmegnagyobbodás esetén előforduló szőrös sejtes leukémia eseteinek 75%-ában a splenectomia hatásos. Ha a citopéniához nem társul lép megnagyobbodás, vagy egyéb szervi elváltozások vagy lymphadenopathia áll fenn, akkor a választandó kezelés alfa-interferon alkalmazása (3 000 000-9 000 000 egység naponta intramuszkulárisan, hosszú hónapokon keresztül, figyelembe véve a vérkép pozitív dinamikáját , elváltozások az érintett szövetekben).Külön formája a bőrelváltozásokkal járó krónikus limfocitás leukémia - a Sezary forma A folyamat gyakran bőrelváltozásokkal, bőrviszketéssel, helyi nyirokrendszeri infiltrátumok megjelenésével kezdődik az epidermisz alatt, amelyek aztán teljessé válhatnak. Limfocitózis és a vérben fokozatosan növekszik a hibás limfociták százalékos aránya.Ezek általában nagyméretű, hurkos szerkezetű sejtmag körvonalakkal rendelkező sejtek, de a sejtek lehetnek kicsik is bab alakú maggal.Ezek a limfociták bizonyítottan a A nyirokcsomók vegyes természetűek lehetnek: egyes nyirokcsomók a bőr fertőzése miatt reaktívan, mások leukémiás beszűrődésük következtében megnagyobbodnak.A lép megnagyobbodhat a betegség lefolyása során. A Sezari-forma kezelésében gyakran hatásos kis dózisú klorobutin (napi 2-4 mg/nap több hónapig, vérvizsgálat, elsősorban vérlemezkeszint ellenőrzése mellett - 2-3 hetente egyszer) hosszú távú alkalmazása. amely enyhíti viszkető bőr, csökkenti a bőr leukémiás beszűrődését. A leukocitózis növekedésében és a mérsékelt limfadenopátiában megnyilvánuló krónikus limfocitás leukémia kezelése klórbutin alkalmazásával kezdődik. Nagy nyirokcsomók esetén ciklofoszfamidot használnak. A szteroid terápiát autoimmun szövődményekre írják fel, hemorrhagiás szindróma, valamint bizonyos citosztatikumok hatástalansága (ez utóbbi esetben a klorobutint vagy a ciklofoszfamidot néha prednizolonnal kombinálják). A szteroidok hosszú távú alkalmazása krónikus limfocitás leukémiában ellenjavallt. Ha jelentős a perifériás nyirokcsomók sűrűsége vagy a hasi nyirokcsomók érintettsége van a folyamatban, akkor olyan gyógyszerek kombinációit, mint a VAMP, vagy ciklofoszfamid, vinkrisztin vagy vinblasztin és prednizolon (COP vagy CVP) kombinációját sikeresen alkalmazzák. A lépet, a nyirokcsomókat és a bőrt besugározzák. Krónikus limfocitás leukémiában az autoimmun citopenia kezelésének egyik módszere a splenectomia. Különösen fontos a fertőző szövődmények kezelése. BAN BEN Utóbbi időben a leukocytopheresist kezdték alkalmazni magas leukocitózissal és citopéniával járó limfocitás leukémia kezelésére. Beteg krónikus limfocitás leukémia Jó egészségi állapotot és munkaképességet évekig megőriznek. A krónikus monolitikus leukémia a leukémia ritka formája, amelyet a perifériás vér magas monocitózisa (20-40%) jellemez, normál vagy enyhén megnövekedett leukocitaszámmal. Az érett monociták mellett egyetlen promonociták is vannak a vérben. A csontvelőben a monociták százalékos aránya kissé megnövekedett, de a trepanátban a csontvelő-szövet hiperpláziája van, a monocitikus elemek diffúz proliferációjával. A vérben és a vizeletben magas a lizozim tartalma. A betegek 50%-ában a lép tapintható. A krónikus monocitás leukémia hosszú távú kedvező lefolyását felválthatja a terminális stádium, amelynek jellemzői megegyeznek a terminális szakaszok krónikus mieloid leukémia. Előrehaladott stádiumban a folyamat nem igényel speciális kezelést, csak mély vérszegénység esetén szükséges időszakos transzfúzió vörösvérsejt tömeg ami ambulánsan végezhető.

MKB 10 ill nemzetközi osztályozás A 10. összehívás összes betegségéből az ismert kórképek szinte minden rövid megnevezését tartalmazza, beleértve az onkológiaiakat is. A leukémiának röviden az ICD 10 szerint két pontos kódolása van:

  • S91- Limfoid forma.
  • S92— Mieloid forma vagy mieloid leukémia.

De figyelembe kell vennie a betegség természetét is. A kijelöléshez egy alcsoportot használunk, amelyet a pont után írunk.

Limfocita leukémia

KódolásLimfoid leukémia
C 91,0 Akut limfoblaszt leukémia T vagy B prekurzor sejtekkel.
C 91,1 Lymphoplasmatikus forma, Richter-szindróma.
C 91,2 Szubakut limfocitás (jelenleg nem használt kód)
C 91,3 Prolimfocita B-sejt
C 91,4 Szőrös sejt és leukémiás reticuloendotheliosis
C 91,5 T-sejtes limfóma vagy felnőtt leukémia HTLV-1-hez kapcsolódó paraméterrel. Opciók: parázsló, akut, lymphomatoid, parázsló.
C 91,6 Prolimfocita T-sejt
C 91,7 Krónikus nagy szemcsés limfociták.
C 91,8 Érett B-sejt (Burkitt)
C 91,9 Finomítatlan forma.

Mieloid leukémia

Tartalmazza a granulocitást és a mielogént.

KódokMieloid leukémia
C 92,0 Alacsony differenciálódási indexű akut mieloid leukémia (AML), valamint érő forma. (AML1/ETO, AML M0, AML M1, AML M2, AML t-vel (8 ; 21), AML (FAB besorolás nélkül) NOS)
92,1-től Krónikus forma (CML), BCR/ABL-pozitív. A Philadelphia kromoszóma (Ph1) pozitív. t(9:22) (q34;q11). Robbanásválsággal. Kivételek: nem osztályozott mieloproliferatív rendellenesség; atipikus, BCR/ABL negatív; Krónikus myelomonocytás leukémia.
C 92,2 Atipikus krónikus, BCR/ABL negatív.
92,3-tól Mieloid szarkóma, amelyben a daganat éretlen atipikus meleoid sejtekből áll. Ide tartozik a granulocitás szarkóma és a kloroma is.
C 92,4 Akut promyelocyta leukémia paraméterekkel: AML M3 és AML M3 t-vel (15; 17).
92,5-től Akut myelomonocyticus AML M4 és AML M4 Eo paraméterekkel inv (16) vagy t(16;16)
C 92,6 11q23 anomáliával és MLL kromoszóma variációval.
92,7-től Egyéb formák. A kivétel a hipereozinofil szindróma vagy a krónikus eozinofil szindróma.
C 92,8 Multilineáris diszpláziával.
92,9-től Finomítatlan formák.

Okoz

Ne feledjük, hogy a vérrák kialakulásának pontos oka nem ismert. Ezért olyan nehéz az orvosok számára a betegség elleni küzdelem és a megelőzés. De számos olyan tényező van, amely növelheti a vörös folyadékrák esélyét.

  • Fokozott sugárzás
  • Ökológia.
  • Szegényes táplálkozás.
  • Elhízottság.
  • A gyógyszerek túlzott használata.
  • Túlsúly.
  • Dohányzás, alkohol.
  • Káros munka peszticidekkel és vegyi anyagokkal, amelyek befolyásolhatják a vérképzőszervi működést.


Tünetek és rendellenességek

  • A vérszegénység a vörösvértestek elnyomása következtében alakul ki, ami miatt az oxigén nem jut el teljes mértékben az egészséges sejtekhez.
  • Súlyos és gyakori fejfájás. A 3. szakasztól kezdődik, amikor a mérgezés miatt rosszindulatú daganat. Előrehaladott vérszegénység következménye is lehet.
  • Állandó megfázás és fertőző és vírusos betegségek hosszú időszakkal. Ez akkor fordul elő, amikor az egészséges fehérvérsejteket atipikus fehérvérsejtekkel helyettesítik. Nem látják el funkciójukat, és a szervezet kevésbé lesz védett.
  • Ízületi fájdalom és fáradtság.
  • Gyengeség, fáradtság, álmosság.
  • Szisztematikus alacsony fokú láz ok nélkül.
  • Változás az illatokban, ízekben.
  • Fogyás és étvágycsökkenés.
  • Elhúzódó vérzés a vérlemezkék számának csökkenésével a vérben.
  • A nyirokcsomók fájdalma és gyulladása az egész testben.

Diagnosztika

Pontos diagnózist csak alapos vizsgálat és bizonyos vizsgálatok listája után lehet felállítani. Leggyakrabban az embereket rendellenes mutatók biokémiai és általános elemzés vér.

A pontosabb diagnózis érdekében a medencecsontból csontvelő-punkciót végeznek. A sejteket később biopsziára küldik. Az onkológus a test teljes vizsgálatát is elvégzi: MRI, ultrahang, CT, röntgen, a metasztázisok azonosítására.

Kezelés, terápia és prognózis

A kezelés fő típusa a kemoterápia, ahol kémiai mérgeket fecskendeznek a vérbe, amelyek célja a kóros vérsejtek elpusztítása. Az ilyen típusú kezelések veszélye és hatástalansága, hogy az egészséges vérsejtek is elpusztulnak, amelyekből amúgy is kevés van.

Ha elsődleges fókuszt azonosítanak, az orvos kemoterápiát írhat elő, hogy teljesen elpusztítsa a csontvelőt ezen a területen. Az eljárás után besugárzás is végezhető a maradványok elpusztítására. rákos sejtek. A folyamat magában foglalja a donorból származó őssejtek átültetését.

LEUKÉMIA

    Akut leukémia.

    Krónikus limfocitás leukémia.

    Krónikus mieloid leukémia.

    Polycythemia vera.

AKUT LEUKÉMIA

Meghatározás.

Az akut leukémia mieloproliferatív daganat, amelynek szubsztrátja olyan blastok, amelyek nem képesek érett vérsejtekké differenciálódni.

ICD10: C91.0 – Akut limfoblasztos leukémia.

C92.0 – Akut mieloid leukémia.

C93.0 – Akut monocitás leukémia.

Etiológia.

A lappangó vírusfertőzés, a hajlamosító öröklődés és az ionizáló sugárzásnak való kitettség szomatikus mutációkat okozhat a vérképző szövetben. Az őssejthez közeli mutáns pluripotens sejtek között olyan klón képződhet, amely érzéketlen az immunszabályozó hatásokra. A mutáns klónból azonos típusú blasztokból álló daganat képződik, amely a csontvelőn túl is intenzíven szaporodik és metasztatizál. Megkülönböztető tulajdonság tumor blasztok képtelensége tovább differenciálódni érett vérsejtekké.

Patogenezis.

Az akut leukémia patogenezisében a legfontosabb láncszem a normál hematopoietikus szövet funkcionális aktivitásának abnormális blastok általi kompetitív metabolikus elnyomása és a csontvelőből való kiszorítása. Ennek eredményeként aplasztikus vérszegénység, agranulocitózis, thrombocytopenia jellegzetes hemorrhagiás szindrómával, súlyos fertőző szövődmények lépnek fel az immunrendszer minden részének mélyreható rendellenességei miatt, valamint mélyreható degeneratív elváltozások a belső szervek szöveteiben.

A FAB osztályozás (francia, amerikai és brit hematológusok együttműködési csoportja, 1990) szerint a következők:

    Akut limfoblasztos (limfoid) leukémia.

    Akut nem limfoblasztos (mieloid) leukémia.

Az akut limfoblasztos leukémia három típusra osztható:

    L1 - akut mikrolimfoblasztikus típus. A blasztok antigén markerei a limfopoiesis null („sem T, sem B”) vagy thymus-dependens (T) vonalainak felelnek meg. Főleg gyermekeknél fordul elő.

    L2 - akut limfoblasztos. Szubsztrátja tipikus limfoblasztok, melyek antigén markerei megegyeznek az L1 típusú akut leukémia esetében tapasztaltakkal. Felnőtteknél gyakoribb.

    L3 - akut makrolimfocitás és prolimfocitás leukémia. A blastok a B-limfociták antigén markereivel rendelkeznek, és morfológiailag hasonlóak a Burkitt limfóma sejtekhez. Ez a típus ritka. Nagyon rossz prognózisa van.

Az akut nem limfoblasztos (mieloid) leukémiákat 6 típusra osztják:

    M0 - akut, differenciálatlan leukémia.

    M1 - akut myeloblastos leukémia sejtérés nélkül.

    M2 - akut myeloblastos leukémia a sejtérés jeleivel.

    M3 - akut promielocitás leukémia.

    M4 - akut myelomonoblastos leukémia.

    M5 - akut monoblasztikus leukémia.

    M6 - akut erythromyelosis.

Klinikai kép.

Az akut leukémia klinikai lefolyása során a következő szakaszokat különböztetjük meg:

Kezdeti időszak (elsődleges aktív szakasz).

A legtöbb esetben akutan kezdődik, gyakran „influenza” formájában. A testhőmérséklet hirtelen megemelkedik, hidegrázás, torokfájás, ízületi fájdalom és súlyos általános gyengeség jelentkezik. Ritkábban a betegség először thrombocytopeniás purpurában, visszatérő orr-, méh- és gyomorvérzésben nyilvánulhat meg. Néha az akut betegség a beteg állapotának fokozatos romlásával, enyhe ízületi fájdalom, csontfájdalom és vérzés megjelenésével kezdődik. Elszigetelt esetekben a betegség tünetmentes megjelenése lehetséges.

Sok betegnél az akut betegség kezdeti szakaszában a perifériás nyirokcsomók megnagyobbodása és mérsékelt lépmegnagyobbodás észlelhető.

Előrehaladott klinikai és hematológiai megnyilvánulások stádiuma (első roham).

A betegek általános állapotának éles romlása jellemzi. Jellemzőek a súlyos általános gyengeség panaszai, magas láz, csontfájdalom, bal hypochondrium a lép területén, vérzés. Ebben a szakaszban az OL-re jellemző klinikai szindrómák alakulnak ki:

Hiperplasztikus (infiltratív) szindróma.

A nyirokcsomók és a lép megnagyobbodása a leukémiás daganatok terjedésének egyik legjellemzőbb megnyilvánulása. A leukémiás beszűrődés gyakran subkapszuláris vérzést, infarktust és léprepedést okoz.

A máj és a vese is megnagyobbodik a leukémiás beszűrődés miatt. A tüdőben, a mellhártyában és a mediastinalis nyirokcsomókban kialakuló leukémiás szűrlet tüdőgyulladás és exudatív mellhártyagyulladás tüneteiként nyilvánul meg.

Az íny leukémiás beszűrődése duzzanatával, hiperémiájával és fekélyeivel az akut monocitás leukémia gyakori előfordulása.

A bőrben, a szemgolyókban és más helyeken lokalizált daganattömegek (leukemidek) a leukémia nem limfoblasztos (mieloid) formáiban fordulnak elő a betegség későbbi szakaszaiban. Egyes mieloblasztos leukémiákban a leukemidek zöldes színűek ("kloroma") lehetnek a myeloperoxidáz jelenléte miatt a tumor blast sejtekben.

Anémiás szindróma.

A leukémiás infiltráció és a normál csontvelői vérképzés metabolikus gátlása aplasztikus anémia kialakulásához vezet. A vérszegénység általában normokróm. Akut erythromyelosisban hiperkróm megaloblasztoid jellegű lehet, mérsékelten kifejezett hemolitikus komponenssel. Súlyos splenomegalia esetén hemolitikus vérszegénység léphet fel.

Hemorrhagiás szindróma.

Thrombocytopenia, DIC szindróma okozza. Szubkután vérzésként (trombocitopéniás purpura), fogínyvérzésként, orrvérzésként és méhvérzésként nyilvánul meg. Emésztőrendszeri és tüdővérzés, durva hematuria lehetséges. A vérzések mellett gyakran előfordul thrombophlebitis, thromboembolia és egyéb hiperkoagulációs rendellenességek, amelyeket a disszeminált intravaszkuláris koagulációs szindróma okoz. Ez az akut promielocitás és myelomonoblastos leukémia egyik jellegzetes megnyilvánulása.

Immunhiányos szindróma.

Az immunhiányos állapot kialakulását az okozza, hogy az immunkompetens sejtek normál klónjait kiszorítják a csontvelőből leukémiás blasztok. Klinikailag lázban nyilvánul meg, gyakran hektikus típusú. Különböző lokalizációjú krónikus fertőzés gócai jelennek meg. Jellemző a fekélyes necroticus mandulagyulladás, peritonsillaris tályogok, nekrotizáló fogínygyulladás, szájgyulladás, pyoderma, pararectalis tályogok, tüdőgyulladás, pyelonephritis előfordulása. A fertőzés általánossá válása szepszis kialakulásával, többszörös tályogok a májban, vesékben, hemolitikus sárgaság, DIC-szindróma gyakran a beteg halálának oka.

Neuroleukémia szindróma.

Jellemzője a blast proliferációs gócok áttétes terjedése az agyhártyákba, az agyba, a gerincvelői struktúrákba és az idegtörzsekbe. Meningealis tünetekkel nyilvánul meg - fejfájás, hányinger, hányás, homályos látás, nyakmerevség. Az agyban kialakuló nagyméretű daganatszerű leukémiás infiltrátumok gócos tünetekkel és agyidegbénulással járnak.

A kezelés eredményeként remisszió érhető el.

A kezelés hatására a betegség összes klinikai megnyilvánulása kihal (nem teljes remisszió) vagy akár teljes eltűnése (teljes remisszió) következik be.

Visszaesés (második és azt követő rohamok).

A folyamatos mutációk következtében kialakul a tumorblasztok klónja, amely képes „kikerülni” a fenntartó kezelésre használt citosztatikus gyógyszerek hatását. A betegség súlyosbodása az összes jellemző tünetegyüttes visszatérésével következik be az OA előrehaladott klinikai és hematológiai megnyilvánulásainak szakaszai.

A relapszus elleni terápia hatására a remisszió ismét elérhető. Az optimális kezelési taktika gyógyuláshoz vezethet. Ha érzéketlen a kezelésre, az OA terminális stádiumba lép.

Felépülés.

A beteg akkor tekinthető gyógyultnak, ha a teljes klinikai és hematológiai remisszió több mint 5 évig fennáll.

Terminál szakasz.

A leukémiás tumorklón proliferációja és metasztázisa feletti terápiás kontroll elégtelen vagy teljes hiánya jellemzi. A csontvelő és a belső szervek leukémiás blasztok általi diffúz infiltrációja következtében a normál vérképző rendszer teljesen elnyomódik, a fertőző immunitás megszűnik, és mélyreható zavarok lépnek fel a vérzéscsillapító rendszerben. A halál elterjedt fertőző elváltozások, kezelhetetlen vérzés és súlyos mérgezés következtében következik be.

Az akut leukémia morfológiai típusainak klinikai jellemzői.

Akut, differenciálatlan leukémia (M0). Ritkán látható. Nagyon gyorsan fejlődik súlyos aplasztikus anémia és súlyos hemorrhagiás szindróma súlyosbodásával. Remisszió ritkán érhető el. Az átlagos várható élettartam kevesebb, mint 1 év.

Akut myeloblastos leukémia (M1-M2). Az akut nem limfoblasztos leukémia leggyakoribb típusa. A felnőttek gyakrabban betegek. Súlyos, tartósan progresszív lefolyása jellemzi, kifejezett anémiás, vérzéses és immunszuppresszív szindrómákkal. Jellemzőek a bőr és a nyálkahártyák fekélyes-nekrotikus elváltozásai. A betegek 60-80% -ánál remisszió érhető el. Az átlagos várható élettartam körülbelül 1 év.

Akut promielocitás leukémia (M3). Az egyik legrosszindulatúbb változat. Súlyos hemorrhagiás szindróma jellemzi, amely leggyakrabban a beteg halálához vezet. Az erőszakos vérzéses megnyilvánulások a DIC-szindrómához kapcsolódnak, amelynek oka a leukémiás promyelociták tromboplasztin aktivitásának növekedése. Felületük és citoplazmájuk 10-15-ször több tromboplasztint tartalmaz, mint a normál sejtek. Az időben történő kezelés szinte minden második betegnél lehetővé teszi a remisszió elérését. Az átlagos várható élettartam eléri a 2 évet.

Akut myelomonoblastos leukémia (M4). A betegség ezen formájának klinikai tünetei közel állnak az akut myeloblastos leukémiához. A különbségek a nekrózisra való nagyobb hajlam. A DIC-szindróma gyakrabban fordul elő. Minden tizedik beteg neuroleukémiás. A betegség gyorsan fejlődik. Gyakran előfordulnak súlyos fertőző szövődmények. Az átlagos várható élettartam és a tartós remissziók gyakorisága kétszer kisebb, mint az akut myeloblastos leukémia esetében.

Akut monoblasztikus leukémia (M5). Ritka forma. A klinikai megnyilvánulások alig különböznek a myelomonoblastos leukémiától. A gyors és tartós progresszióra való nagyobb hajlam jellemzi. Ezért a leukémia ezen formájában szenvedő betegek átlagos várható élettartama még rövidebb - körülbelül 9 hónap.

Akut erythromyelosis (M6). Ritka forma. Ennek a formának a megkülönböztető jellemzője a tartós, mély vérszegénység. Hiperkróm vérszegénység enyhe hemolízis tüneteivel. A leukémiás eritroblasztokban megaloblasztoid rendellenességeket észlelnek. Az akut erythromyelosis legtöbb esete rezisztens a terápiára. A betegek várható élettartama ritkán haladja meg a 7 hónapot.

Akut limfoblaszt leukémia (L1, L2, L3). Ezt a formát mérsékelten progresszív lefolyás jellemzi. A perifériás nyirokcsomók, a lép és a máj megnagyobbodása kíséri. A hemorrhagiás szindróma és a fekélyes-nekrotikus szövődmények ritkák. Az akut limfoblaszt leukémia várható élettartama 1,5-3 év.

Rövid leírás

Az akut leukémia a hematopoietikus rendszer rosszindulatú betegsége; morfológiai szubsztrát - blastsejtek.

Frekvencia. A férfiaknál 13,2, a nőknél 7,7 eset/lakosság.

A FAB besorolás (franko-amerikai-brit) a leukémiás sejtek morfológiáján alapul (a sejtmag szerkezete, a sejtmag és a citoplazma méretének aránya) Akut myeloblastos (nem limfoblasztos) leukémia (AML) M0 - sejtérés nélkül, myelogen differenciálódás csak immunológiailag bizonyított M1 - sejtérés nélkül M2 - AML sejtdifferenciálódással, M3 - promyelocyta M4 - myelomonocytás M5 - monoblaszt leukémia M6 - eritroblaszt leukémia M7 - megakarioblaszt leukémia Akut limfoblaszt leukémia (ALL): L1 - sejtkülönbség nélkül homogén sejtek) L2 - sejtdifferenciálódással (morfológiailag heterogén sejtpopuláció) L3 - Burkett-szerű leukémiák Differenciálatlan leukémia - ebbe a kategóriába tartoznak azok a leukémiák, amelyek sejtjei nem azonosíthatók mieloblasztosnak vagy limfoblasztosnak (sem kémiai, sem immunológiai módszerekkel) Myelopoieticus dysplasia Refrakter anaemia anélkül (a csontvelőben blastok és promyelociták vannak<10%) Рефрактерная анемия с бластозом (в костном мозге бласты и промиелоциты 10 30%) Рефрактерная анемия с избытком бластов в трансформации Хронический миеломоноцитарный лейкоз.

VALÓDI osztályozás (Revised European American Classification of Lymphoid neoplasms), felülvizsgált (Európa-amerikai) limfoid hemoblasztózisok osztályozása Pre B sejt tumorok Pre B limfoblaszt leukémia/limfóma Pre T-sejtes tumorok Pre T limfoblasztos leukémia/limfóma Perifériás B-sejtes daganatok krónikus limfoid leukémia/kisméretű limfóma limfociták Lymphoplasmacyticus limfóma Köpenysejtes limfóma Follikuláris limfóma Marginális zóna sejt limfóma Szőrös sejt leukémia Plazmacitóma/plazmocitás mielóma Diffúz nagy limfocita limfóma Burkett limfóma Perifériás T-sejtes és NK-sejtes daganatok T-sejtes krónikus limfocitás leukémia funcidák T-sejtes lymphocytás leukémia funcidák T-sejtes limfocitás leukémia és nagy szemcsés limfóma S-limfózis Angioimmunoblasztos T-sejtes limfóma Angiocentrikus limfóma (NK- és T-sejtes limfóma) Bél T-sejtes limfóma Leukémia/felnőtt T-sejtes limfóma Anaplasztikus nagysejtes limfóma

Az AML változatai (WHO osztályozás, 1999) AML t(8;21)(q22;q22) AML t(15;17) (q22;q11 12) Akut mielomonoblasztos leukémia AML kóros csontvelőeozinofíliával (inv(16) (p13q22) ) vagy t(16;16) (p13;q11) AML 11q23 (MLL) defektussal Akut eritroid leukémia Akut megakaryocytás leukémia Akut basofil leukémia Akut panmyelosis myelofibrosissal Akut panmyelosis myelofibrosissal Akut AMLplatikus bifenotyás AMLplaágia második multilineáris leukémia.

Immunhisztokémiai vizsgálat (celluláris fenotípus meghatározása) szükséges a leukémia immunológiai változatának tisztázásához, amely befolyásolja a kezelési rendet és a klinikai prognózist

Akut limfoblasztos leukémia (247640, , szomatikus sejtek mutációja) - az esetek 85%-a, az összes gyermekkori leukémia 90%-át teszi ki, felnőtteknél meglehetősen ritkán alakul ki. Citokémiai reakciók: pozitív a terminális dezoxinukleotidil-transzferázra; negatív myeloperoxidosisra, glikogénre. A sejtmembrán markerek használata lehetővé tette a B altípusok azonosítását - sejt - az esetek 75%-a Rozettaképződés hiányában T - sejt Egyéb lehetőségek (ritka). Az altípusok differenciáldiagnózisa fontos a prognózis szempontjából, mert A T-sejtes változatokat nehéz kezelni.

Az akut myeloid leukémia gyakrabban fordul elő felnőtteknél, és az altípus a sejtdifferenciálódás szintjétől függ. A legtöbb esetben a mieloblaszt klón hematopoietikus őssejtekből származik, amelyek többszörösen képesek granulociták, eritrociták, makrofágok vagy megakariociták kolóniaképző egységeivé differenciálódni, ezért a legtöbb betegnél a rosszindulatú klónok nem mutatják limfoid vagy eritroid vonalak jeleit. Az AML a leginkább gyakran megfigyelhető; négy változata van (M0 - M3) M0 és M1 - akut leukémia sejtdifferenciálódás nélkül M2 - akut sejtdifferenciálódással M3 - promielocitás leukémia, amelyet abnormális promyelociták jelenléte jellemez óriási granulátumokkal; gyakran kombinálják a granulátum tromboplasztikus hatása által okozott DIC-vel, ami megkérdőjelezi a heparin terápia célszerűségét. Az M3 prognózisa kedvezőtlenebb, mint az M0–M1 esetében A myelomonoblastos és monoblasztos leukémiát (M4 és M5) a nem eritroid sejtek, például a monoblasztok túlsúlya jellemzi. Az M4 és M5 az összes AML eset 5-10%-át teszik ki. Gyakori tünet a májban, a lépben, az ínyben és a bőrben kialakuló velőn kívüli hematopoiesis gócok, az 50-100109/l-t meghaladó hyperleukocytosis. A terápia iránti érzékenység és a túlélés alacsonyabb, mint az eritroleukémia (M6) más típusú akut mieloblasztos leukémiája esetén. Az akut myeloblastos leukémia egy változata, amelyet az eritroid prekurzorok fokozott proliferációja kísér; abnormális, magvú vörösvértestek jelenléte jellemzi. Az eritroleukémia kezelésének hatékonysága hasonló a más altípusok kezelésének eredményéhez, vagy valamivel alacsonyabb.A megakarioblasztos leukémia (M7) egy ritka változata, amelyet csontvelőfibrózissal (akut myelosclerosis) kombinálnak. Nem reagál jól a terápiára. A prognózis kedvezőtlen.

A patogenezist a daganatsejtek csontvelőben történő elszaporodása és különböző szervekbe történő áttétje okozza. A normál hematopoiesis gátlása két fő tényezővel függ össze: a normál vérképzőcsíra károsodása és kiszorítása a rosszul differenciált leukémiás sejtek által; a blasztsejtek által termelt inhibitorok, amelyek elnyomják a normál hematopoietikus sejtek növekedését.

Az akut leukémia stádiumai Primer - aktív fázis Remisszió (kezeléssel) - teljes klinikai - hematológiai A blasztok tartalma a csontvelőben 5% alatti normál cellularitással A klinikai képen nincs proliferatív szindróma Relapszus (korai és késői) Izolált csont velő - a csontvelő blaszttartalma több mint 25 % Extramarrow Neuroleukémia (neurológiai tünetek, 10 sejtnél több sejt citózisa, blasztok a cerebrospinális folyadékban) Herék (egy vagy két here méretének növekedése, blasztok jelenléte citológiai és szövettani vizsgálatok igazolják) Vegyes terminális fázis (kezelés hiányában és terápiával szembeni rezisztencia esetén)

Tünetek (jelek)

Az akut leukémia klinikai képét a csontvelő blastsejtek általi beszűrődésének mértéke és a vérképző hajtások gátlása határozza meg Csontvelői vérképzés gátlása Anémiás szindróma (myelophthisic anaemia) Hemorrhagiás szindróma (thrombocytopenia miatt bőrvérzések nem fordulnak elő - petechiák, ekchymózisok, nyálkahártyák vérzése - orrvérzés, belső vérzés) fertőzések (leukociták diszfunkciója) limfoproliferatív szindróma Hepatosplenomegalia megnagyobbodott nyirokcsomók hyperplasiás szindróma Csontfájdalom Bőr (leukémiák), agyhártya (neuroleukémia) és belső szervek elváltozásai Láz Hyperhidrosis Súlyos gyengeség.

Diagnosztika

Az akut leukémia diagnózisát a csontvelőben lévő blastok igazolják. A leukémia altípusának azonosítására hisztokémiai, immunológiai és citogenetikai kutatási módszereket alkalmaznak.

Laboratóriumi vizsgálatok A perifériás vérben a leukociták szintje a súlyos leukopeniától (2,0109/l alatt) a hyperleukocytosisig változhat; vérszegénység, thrombocytopenia; blastos sejtek jelenléte a teljes blastosisig A sejtek felgyorsult életciklusa miatti hiperurikémia Hypofibrinogenemia és az egyidejű DIC miatt megnövekedett fibrinpusztító termékek tartalma. A drogok hatása. GC-t nem szabad felírni a végleges diagnózis felállításáig. A blastsejtek prednizolonnal szembeni nagy érzékenysége pusztulásukhoz és átalakulásukhoz vezet, ami megnehezíti a diagnózist.

A kezelés összetett; a cél a teljes remisszió elérése. Jelenleg a hematológiai centrumok különféle kemoterápiás protokollokat alkalmaznak, amelyek a polikemoterápia és a kezelés intenzifikálása elvein alapulnak.

A kemoterápia több szakaszból áll: Remisszió indukálása ALL esetén - a következő sémák egyike: vinkrisztin kombinációi intravénásan hetente, prednizolon orálisan naponta, daunorubicin és aszparagináz 1-2 hónapig folyamatosan AML esetén - citarabin kombinációja intravénásan vagy szubkután, intravénásan daunorubicin , néha tioguaninnal kombinálva. Intenzívebb posztindukciós kemoterápia, amely elpusztítja a megmaradt leukémiás sejteket, növeli a remisszió időtartamát A remisszió megszilárdítása: a szisztémás kemoterápia folytatása és a neuroleukémia megelőzése ALL-ben (metotrexát endolumbális beadása ALL-ben sugárterápiával kombinálva a gerincvelői agyba bevonása) Fenntartó terápia: időszakos remissziós reindukciós kúrák .

Az AML M3-at retinsavval (tretinoin) kezelik.

A csontvelő-transzplantáció a választott módszer az akut myeloblastos leukémia és az összes akut leukémia visszaesése esetén. A transzplantáció fő feltétele a teljes klinikai és hematológiai remisszió (a csontvelő blasztjainak tartalma kevesebb, mint 5%, abszolút limfocitózis hiánya). A műtét előtt a kemoterápia ultra-nagy dózisban, önmagában vagy sugárterápiával kombinálva (a leukémiás sejtek teljes elpusztítása érdekében) adható.Az optimális donor egy egypetéjű iker vagy testvér; Gyakrabban olyan donorokat használnak, amelyek 35%-os Ag HLA egyezést mutatnak. Kompatibilis donorok hiányában a remisszió időszakában vett csontvelő autotransplantációt alkalmazzák, melynek fő szövődménye a graft-versus-host betegség. A donor T-limfociták transzplantációja következtében alakul ki, amelyek idegenként ismerik fel a recipiens Ag-jeit és immunreakciót váltanak ki ellenük. Az akut reakció a transzplantáció után 20-100 napon belül, késleltetett reakció 6-12 hónap múlva alakul ki.A fő célszervek a bőr (dermatitis), a gyomor-bél traktus (hasmenés) és a máj (toxikus hepatitis) A kezelés hosszú távú , általában a prednizolon, ciklosporin és kis dózisú azathioprin kombinációjának felírására korlátozódik A transzplantáció utáni időszak lefolyását az előkészítő kezelési rendek, az intersticiális tüdőgyulladás kialakulása és a graft kilökődése (ritkán) is befolyásolják.

Helyettesítő terápia Vörösvérsejtek transzfúziója a Hb-szint 100 g/l alatti szinten tartására. Transzfúziós állapotok: nem rokon donor, leukocita szűrők használata Friss vérlemezkék transzfúziója (csökkenti a vérzés kockázatát). Javallatok: thrombocytaszám kevesebb, mint 20109/l; hemorrhagiás szindróma 50109/l alatti vérlemezke-tartalommal.

A fertőzések megelőzése a kemoterápia következtében neutropeniában szenvedő betegek túlélésének fő feltétele A beteg teljes elkülönítése Szigorú egészségügyi és fertőtlenítési rend - gyakori nedves tisztítás (legfeljebb napi 4-5 alkalommal), a helyiségek szellőztetése és kvarcozása; eldobható eszközök használata, steril ruházat az egészségügyi személyzet számára Antibiotikumok, gombaellenes és vírusellenes szerek megelőző alkalmazása (ha a szegmentált neutrofil tartalom 0,5109/l alatt van, a Pneumocystis pneumonia profilaxisa javallott) Testhőmérséklet emelkedése esetén klinikai és bakteriológiai vizsgálatokat végeznek, és azonnal megkezdik a széles spektrumú baktericid antibiotikum-kombinációkkal történő kezelést: cefalosporinok, aminoglikozidok és félszintetikus penicillinek A széles spektrumú antibiotikumokkal végzett kezelést követően fellépő másodlagos testhőmérséklet-emelkedéshez empirikusan alkalmaznak gombaellenes szereket (amfotericin B). Kolóniastimuláló faktorok (például molgramosztim) felírhatók a neutropenia megelőzésére és kezelésére.

Prognózis Az akut limfocitás leukémiában szenvedő gyermekek prognózisa kedvező: 95%-uk vagy annál több teljes remissziót tapasztal. A betegek 70-80%-ánál 5 évig nem jelentkezik a betegség, és gyógyultnak tekintik. Ha visszaesés következik be, a legtöbb esetben egy második teljes remisszió is elérhető. A második remisszióban lévő betegek csontvelő-transzplantációra jelöltek, a hosszú távú túlélés valószínűsége 35-65%, az akut myeloblastos leukémiában szenvedő betegek prognózisa kedvezőtlen. A modern kemoterápiás sémákkal megfelelő kezelésben részesülő betegek 75%-a teljes remissziót ér el, a betegek 25%-a meghal (a remisszió időtartama 12-18 hónap). Az esetek 20%-ában gyógyulásról számoltak be, ha a remissziót követően folytatják az intenzív kezelést. Az AML M3 variánsának prognózisa javul a retinsav gyógyszerekkel végzett kezeléssel. A 30 év alatti betegek az első teljes remisszió elérése után csontvelő-transzplantáción eshetnek át. Az allogén transzplantáción átesett fiatal betegek 50%-ánál hosszú távú remisszió alakul ki. Biztató eredmények születtek az autológ csontvelő-transzplantációkkal is.

Gyermekek Az összes akut leukémia 80%-a - MINDEN Kedvezőtlen prognosztikai faktor ALL-ra 1 év alatti és 10 év feletti gyermek életkora Férfi nem T-sejt változata ALL Leukocita tartalom a diagnózis időpontjában több mint 20109/l Klinikai hiánya és hematológiai remisszió az indukció hátterében Prognózis és jelenlegi. A klinikai és hematológiai remisszió 80%-a. 5 éves túlélési arány - 40-50%.

Idős. Csökkentett tolerancia az allogén csontvelővel szemben. A transzplantáció maximális életkora 50 év. Az autológ transzplantáció szervkárosodás és általános szomatikus közérzet hiányában 50 év feletti betegeknél végezhető.

Rövidítések MDS - myelodysplasiás szindróma ALL - akut limfoblasztos leukémia AML - akut myeloid leukémia.

ICD-10 C91.0 Akut limfoblaszt leukémia C92 Myeloid leukémia [mieloid leukémia] C93.0 Akut monocitás leukémia

Mieloid leukémia [mieloid leukémia] (C92)

Tartalmazza: leukémia:

  • granulocita
  • mielogén

Akut mieloid leukémia minimális differenciálódással

Akut myeloblastos leukémia (éréssel)

AML (FAB besorolás nélkül) NOS

Refrakter vérszegénység túlzott blastokkal az átalakulásban

Kivétel: krónikus mieloid leukémia (C92.1) súlyosbodása

Krónikus mieloid leukémia:

  • Philadelphia kromoszóma (Ph1) pozitív
  • t(9:22)(q34; q11)
  • robbanásválsággal

Kizárva:

  • Atípusos krónikus mieloid leukémia, BCR/ABL negatív (C92.2)
  • Krónikus myelomonocytás leukémia (C93.1)
  • nem osztályozott mieloproliferatív rendellenesség (D47.1)

Megjegyzés: éretlen mieloid sejtek daganata.

AML M3 t(15; 17)-vel és változataival

AML M4 Eo inv(16) vagy t(16;16)

Akut mieloid leukémia MLL génvariációval

Nem tartalmazza: krónikus eozinofil leukémia [hipereozinofil szindróma] (D47.5)

Megjegyzés: Akut myeloid leukémia diszpláziával a fennmaradó hematopoiesisben és/vagy a kórelőzményben myelodysplasiás betegségben.

Oroszországban a Betegségek Nemzetközi Osztályozása, 10. revízióját (ICD-10) egységes normatív dokumentumként fogadták el a megbetegedések, az összes osztály egészségügyi intézményeibe tett lakossági látogatások okainak és a halálokok rögzítésére.

Az ICD-10-et az Orosz Föderáció egész területén 1999-ben vezették be az egészségügyi gyakorlatba az Orosz Egészségügyi Minisztérium 1997. május 27-i rendeletével. 170. sz

Az új változat (ICD-11) kiadását a WHO 2017-2018-ra tervezi.

A WHO változtatásaival és kiegészítéseivel.

Változások feldolgozása és fordítása © mkb-10.com

/ Belső betegségek / 8. fejezet LEUKÉMIA-r

Az akut leukémia mieloproliferatív daganat, amelynek szubsztrátja olyan blastok, amelyek nem képesek érett vérsejtekké differenciálódni.

ICD10: C91.0 – Akut limfoblasztos leukémia.

C92.0 – Akut mieloid leukémia.

C93.0 – Akut monocitás leukémia.

A lappangó vírusfertőzés, a hajlamosító öröklődés és az ionizáló sugárzásnak való kitettség szomatikus mutációkat okozhat a vérképző szövetben. Az őssejthez közeli mutáns pluripotens sejtek között olyan klón képződhet, amely érzéketlen az immunszabályozó hatásokra. A mutáns klónból azonos típusú blasztokból álló daganat képződik, amely a csontvelőn túl is intenzíven szaporodik és metasztatizál. A tumorblasztok megkülönböztető jellemzője, hogy képtelenek tovább differenciálódni érett vérsejtekké.

Az akut leukémia patogenezisében a legfontosabb láncszem a normál hematopoietikus szövet funkcionális aktivitásának abnormális blastok általi kompetitív metabolikus elnyomása és a csontvelőből való kiszorítása. Ennek eredményeként aplasztikus vérszegénység, agranulocitózis, thrombocytopenia jellegzetes hemorrhagiás szindrómával, súlyos fertőző szövődmények lépnek fel az immunrendszer minden részének mélyreható rendellenességei miatt, valamint mélyreható degeneratív elváltozások a belső szervek szöveteiben.

A FAB osztályozás (francia, amerikai és brit hematológusok együttműködési csoportja, 1990) szerint a következők:

Akut limfoblasztos (limfoid) leukémia.

Akut nem limfoblasztos (mieloid) leukémia.

Az akut limfoblasztos leukémia három típusra osztható:

L1 - akut mikrolimfoblasztikus típus. A blasztok antigén markerei a limfopoiesis null („sem T, sem B”) vagy thymus-dependens (T) vonalainak felelnek meg. Főleg gyermekeknél fordul elő.

L2 - akut limfoblasztos. Szubsztrátja tipikus limfoblasztok, melyek antigén markerei megegyeznek az L1 típusú akut leukémia esetében tapasztaltakkal. Felnőtteknél gyakoribb.

L3 - akut makrolimfocitás és prolimfocitás leukémia. A blastok a B-limfociták antigén markereivel rendelkeznek, és morfológiailag hasonlóak a Burkitt limfóma sejtekhez. Ez a típus ritka. Nagyon rossz prognózisa van.

Az akut nem limfoblasztos (mieloid) leukémiákat 6 típusra osztják:

M0 - akut, differenciálatlan leukémia.

M1 - akut myeloblastos leukémia sejtérés nélkül.

M2 - akut myeloblastos leukémia a sejtérés jeleivel.

M3 - akut promielocitás leukémia.

M4 - akut myelomonoblastos leukémia.

M5 - akut monoblasztikus leukémia.

M6 - akut erythromyelosis.

Az akut leukémia klinikai lefolyása során a következő szakaszokat különböztetjük meg:

Kezdeti időszak (elsődleges aktív szakasz).

A legtöbb esetben akutan kezdődik, gyakran „influenza” formájában. A testhőmérséklet hirtelen megemelkedik, hidegrázás, torokfájás, ízületi fájdalom és súlyos általános gyengeség jelentkezik. Ritkábban a betegség először thrombocytopeniás purpurában, visszatérő orr-, méh- és gyomorvérzésben nyilvánulhat meg. Néha az akut betegség a beteg állapotának fokozatos romlásával, enyhe ízületi fájdalom, csontfájdalom és vérzés megjelenésével kezdődik. Elszigetelt esetekben a betegség tünetmentes megjelenése lehetséges.

Sok betegnél az akut betegség kezdeti szakaszában a perifériás nyirokcsomók megnagyobbodása és mérsékelt lépmegnagyobbodás észlelhető.

Előrehaladott klinikai és hematológiai megnyilvánulások stádiuma (első roham).

A betegek általános állapotának éles romlása jellemzi. Jellemző panaszok a súlyos általános gyengeség, magas láz, csontfájdalom, a lép környékén a bal hypochondriumban jelentkező fájdalom, vérzés. Ebben a szakaszban az OL-re jellemző klinikai szindrómák alakulnak ki:

Hiperplasztikus (infiltratív) szindróma.

A nyirokcsomók és a lép megnagyobbodása a leukémiás daganatok terjedésének egyik legjellemzőbb megnyilvánulása. A leukémiás beszűrődés gyakran subkapszuláris vérzést, infarktust és léprepedést okoz.

A máj és a vese is megnagyobbodik a leukémiás beszűrődés miatt. A tüdőben, a mellhártyában és a mediastinalis nyirokcsomókban kialakuló leukémiás szűrlet tüdőgyulladás és exudatív mellhártyagyulladás tüneteiként nyilvánul meg.

Az íny leukémiás beszűrődése duzzanatával, hiperémiájával és fekélyeivel az akut monocitás leukémia gyakori előfordulása.

A bőrben, a szemgolyókban és más helyeken lokalizált daganattömegek (leukemidek) a leukémia nem limfoblasztos (mieloid) formáiban fordulnak elő a betegség későbbi szakaszaiban. Egyes mieloblasztos leukémiákban a leukemidek zöldes színűek ("kloroma") lehetnek a myeloperoxidáz jelenléte miatt a tumor blast sejtekben.

A leukémiás infiltráció és a normál csontvelői vérképzés metabolikus gátlása aplasztikus anémia kialakulásához vezet. A vérszegénység általában normokróm. Akut erythromyelosisban hiperkróm megaloblasztoid jellegű lehet, mérsékelten kifejezett hemolitikus komponenssel. Súlyos splenomegalia esetén hemolitikus vérszegénység léphet fel.

Thrombocytopenia, DIC szindróma okozza. Szubkután vérzésként (trombocitopéniás purpura), fogínyvérzésként, orrvérzésként és méhvérzésként nyilvánul meg. Emésztőrendszeri és tüdővérzés, durva hematuria lehetséges. A vérzések mellett gyakran előfordul thrombophlebitis, thromboembolia és egyéb hiperkoagulációs rendellenességek, amelyeket a disszeminált intravaszkuláris koagulációs szindróma okoz. Ez az akut promielocitás és myelomonoblastos leukémia egyik jellegzetes megnyilvánulása.

Az immunhiányos állapot kialakulását az okozza, hogy az immunkompetens sejtek normál klónjait kiszorítják a csontvelőből leukémiás blasztok. Klinikailag lázban nyilvánul meg, gyakran hektikus típusú. Különböző lokalizációjú krónikus fertőzés gócai jelennek meg. Jellemző a fekélyes necroticus mandulagyulladás, peritonsillaris tályogok, nekrotizáló fogínygyulladás, szájgyulladás, pyoderma, pararectalis tályogok, tüdőgyulladás, pyelonephritis előfordulása. A fertőzés általánossá válása szepszis kialakulásával, többszörös tályogok a májban, vesékben, hemolitikus sárgaság, DIC-szindróma gyakran a beteg halálának oka.

Jellemzője a blast proliferációs gócok áttétes terjedése az agyhártyákba, az agyba, a gerincvelői struktúrákba és az idegtörzsekbe. Meningealis tünetekkel nyilvánul meg - fejfájás, hányinger, hányás, homályos látás, nyakmerevség. Az agyban kialakuló nagyméretű daganatszerű leukémiás infiltrátumok gócos tünetekkel és agyidegbénulással járnak.

A kezelés eredményeként remisszió érhető el.

A kezelés hatására a betegség összes klinikai megnyilvánulása kihal (nem teljes remisszió) vagy akár teljes eltűnése (teljes remisszió) következik be.

Visszaesés (második és azt követő rohamok).

A folyamatos mutációk következtében kialakul a tumorblasztok klónja, amely képes „kikerülni” a fenntartó kezelésre használt citosztatikus gyógyszerek hatását. A betegség súlyosbodása az OA előrehaladott klinikai és hematológiai megnyilvánulásainak stádiumára jellemző összes szindróma visszatérésével következik be.

A relapszus elleni terápia hatására a remisszió ismét elérhető. Az optimális kezelési taktika gyógyuláshoz vezethet. Ha érzéketlen a kezelésre, az OA terminális stádiumba lép.

A beteg akkor tekinthető gyógyultnak, ha a teljes klinikai és hematológiai remisszió több mint 5 évig fennáll.

A leukémiás tumorklón proliferációja és metasztázisa feletti terápiás kontroll elégtelen vagy teljes hiánya jellemzi. A csontvelő és a belső szervek leukémiás blasztok általi diffúz infiltrációja következtében a normál vérképző rendszer teljesen elnyomódik, a fertőző immunitás megszűnik, és mélyreható zavarok lépnek fel a vérzéscsillapító rendszerben. A halál elterjedt fertőző elváltozások, kezelhetetlen vérzés és súlyos mérgezés következtében következik be.

Az akut leukémia morfológiai típusainak klinikai jellemzői.

Akut, differenciálatlan leukémia (M0). Ritkán látható. Nagyon gyorsan fejlődik súlyos aplasztikus anémia és súlyos hemorrhagiás szindróma súlyosbodásával. Remisszió ritkán érhető el. Az átlagos várható élettartam kevesebb, mint 1 év.

Akut myeloblastos leukémia (M1-M2). Az akut nem limfoblasztos leukémia leggyakoribb típusa. A felnőttek gyakrabban betegek. Súlyos, tartósan progresszív lefolyása jellemzi, kifejezett anémiás, vérzéses és immunszuppresszív szindrómákkal. Jellemzőek a bőr és a nyálkahártyák fekélyes-nekrotikus elváltozásai. A betegek 60-80% -ánál remisszió érhető el. Az átlagos várható élettartam körülbelül 1 év.

Akut promielocitás leukémia (M3). Az egyik legrosszindulatúbb változat. Súlyos hemorrhagiás szindróma jellemzi, amely leggyakrabban a beteg halálához vezet. Az erőszakos vérzéses megnyilvánulások a DIC-szindrómához kapcsolódnak, amelynek oka a leukémiás promyelociták tromboplasztin aktivitásának növekedése. Felületük és citoplazmájuk sokszor több tromboplasztint tartalmaz, mint a normál sejtekben. Az időben történő kezelés szinte minden második betegnél lehetővé teszi a remisszió elérését. Az átlagos várható élettartam eléri a 2 évet.

Akut myelomonoblastos leukémia (M4). A betegség ezen formájának klinikai tünetei közel állnak az akut myeloblastos leukémiához. A különbségek a nekrózisra való nagyobb hajlam. A DIC-szindróma gyakrabban fordul elő. Minden tizedik beteg neuroleukémiás. A betegség gyorsan fejlődik. Gyakran előfordulnak súlyos fertőző szövődmények. Az átlagos várható élettartam és a tartós remissziók gyakorisága kétszer kisebb, mint az akut myeloblastos leukémia esetében.

Akut monoblasztikus leukémia (M5). Ritka forma. A klinikai megnyilvánulások alig különböznek a myelomonoblastos leukémiától. A gyors és tartós progresszióra való nagyobb hajlam jellemzi. Ezért a leukémia ezen formájában szenvedő betegek átlagos várható élettartama még rövidebb - körülbelül 9 hónap.

Akut erythromyelosis (M6). Ritka forma. Ennek a formának a megkülönböztető jellemzője a tartós, mély vérszegénység. Hiperkróm vérszegénység enyhe hemolízis tüneteivel. A leukémiás eritroblasztokban megaloblasztoid rendellenességeket észlelnek. Az akut erythromyelosis legtöbb esete rezisztens a terápiára. A betegek várható élettartama ritkán haladja meg a 7 hónapot.

Akut limfoblaszt leukémia (L1, L2, L3). Ezt a formát mérsékelten progresszív lefolyás jellemzi. A perifériás nyirokcsomók, a lép és a máj megnagyobbodása kíséri. A hemorrhagiás szindróma és a fekélyes-nekrotikus szövődmények ritkák. Az akut limfoblaszt leukémia várható élettartama 1,5-3 év.

Teljes vérkép: a vörösvértestek, leukociták, vérlemezkék számának csökkenése. A vérszegénység gyakran normocitás, normokróm, de akut erythromyelosisban szenvedő betegeknél macrocytosis és nukleáris formák megjelenése a vérben megaloblastosis jeleivel figyelhető meg. A megaloblaszt-szerű rendellenességek nem tűnnek el cianokobalamin-kezeléssel. A robbanássejteket észlelik. A leukocita képletet a „leukémiás kudarc” jelensége jellemzi - a blasztok és a leukociták érett formáinak jelenléte a közepes differenciálódási fokú sejtek hiányában ("kudarc"). Ez egyidejűleg két szaporodó sejtvonal jelenlétét jelzi. Az egyik vonal normális, érett sejtes formákban végződik. A másik vonal a blastsejtek tumorklónja, amelyek nem képesek további differenciálódásra. A leukociták mennyiségétől és a perifériás vérben lévő blastsejtek számától függően a leukémia három formáját különböztetjük meg: leukémiás - magas leukocitózissal, 100x10 9 /l-ig és nagyszámú blastossággal; subleukémiás, amikor a blasztok száma kissé meghaladja a vér normál leukociták tartalmát; aleukémiás - a perifériás vérben lévő blastok hiányában. Az utóbbi esetben általában pancytopenia figyelhető meg - leukopenia, vérszegénység, thrombocytopenia.

Sternális pont: A kezeletlen betegek csontvelőjében a blasztok az összes magos sejt több mint 50%-át teszik ki. Az eritrocita, granulocita és megakariocita vonalak elnyomódnak. Felfedik a megaloblasztos eritrogenezis jeleit.

Az agy-gerincvelői folyadék vizsgálata: magas citózis, blastsejtek észlelhetők, megnövekedett fehérjetartalom.

A blasztok hisztokémiai vizsgálata: akut myeloid leukémiában a blastsejtek pozitív reakciót adnak mieloperoxidázra, lipidekre, klór-acetát-észterázra, egyes formákban pozitív CHIC reakció lehetséges (akut erythromyelosis); akut limfoblasztos leukémiában a glikogén mindig kimutatható (pozitív CHIC-reakció), de nem reagálnak peroxidázra, lipidekre, klór-acetát-észterázra vagy kationos fehérjékre (katepszinekre).

Leukémiás sejtek immuntipizálása: feltárja, hogy a limfoblasztok a T- vagy B-limfociták populációihoz tartoznak-e, vagy nem meghatározott (sem T, sem B) típusúak. Lehetővé teszi a blasztsejtek (CD-markerek) differenciálódási klaszterek jelenlétének vagy hiányának azonosítását, ami nagy jelentőséggel bír az akut limfoblasztos leukémia és a mieloblasztos leukémia megkülönböztetésének pontos diagnosztizálásában.

Citogenetikai vizsgálat: lehetővé teszi a blastsejtek kromoszóma-rendellenességeinek (aneuploidia, pszeudodiploidia) kimutatását, amelyeket leggyakrabban akut mieloid leukémiában észlelnek - az esetek közel 50% -ában.

Az OL diagnózisának indoklása.

A vérszegénység, vérzéses, immunhiányos szindrómák, meningeális jelenségek formájában jelentkező klinikai megnyilvánulások lehetővé teszik a betegség gyanúját, és okként szolgálnak a szegycsont-punkció végrehajtására. Az OA diagnózisa a csontvelő blastos infiltrációjának kimutatásán alapul a sternális punkció és/vagy a csípőszárny trepanobiopszia során.

A differenciáldiagnózis elsősorban leukemoid reakciókkal, agranulocitózissal, aplasztikus anémiával történik.

Súlyos fertőző betegségekben és rosszindulatú daganatokban szenvedő betegeknél előforduló leukemoid reakciókban kifejezett leukocitózis fordulhat elő, ha a képletet balra tolják, amíg egyedi blastok nem jelennek meg. Azonban az OB-val ellentétben ezekben az állapotokban nincs „leukémiás provol” – a blast és az érett leukocita köztes differenciálódás sejtes formáinak hiánya. A vérszegénység és a thrombocytopenia nem jellemző a leukemoid reakciókra. A csontvelőben és a perifériás vérben nincs jelentős növekedés a blastsejtek mennyiségében.

Amikor toxikus vagy immuntényezők által okozott agranulocitózis jelentkezik, blastsejtek jelennek meg a perifériás vérben. Előfordulhat olyan helyzet, amikor a kenet egyetlen érett leukocitát és köztes sejtformák nélküli blastokat mutat. A vérkenet dinamikus vizsgálata során azonban a blasztokat követő intermedier formák megjelenése figyelhető meg, ami soha nem figyelhető meg AL-ben szenvedő betegeknél. Agranulocitózis esetén, az AL-tól eltérően, a csontvelőben nincs túlzott mennyiségű blast sejt.

Az OB-vel ellentétben az aplasztikus anémiát nem a nyirokcsomók és a lép megnagyobbodása jellemzi. Az OA-val ellentétben aplasztikus anémiában a csontvelő kimerülése és a zsírszövet magas tartalma. A csontvelőben a blastok száma élesen csökken, ami AL-ban nem történik meg.

Általános vérvizsgálat.

A csípőszárny sternális punkciója és/vagy trepanobiopsziája.

A leukémiás limfoblasztok populációjának immuntipizálása (B vagy T).

A blasztok hisztokémiai tipizálása a nem limfoblaszt leukémia morfológiai változatának meghatározására.

Kemoterápiát és csontvelő-transzplantációt alkalmaznak.

Az akut leukémia kemoterápiáját a következő szakaszokban végzik:

Akut mieloid leukémia (akut myeloid leukémia)

Akut mieloid leukémiában a kórosan differenciálódott, hosszú életű mieloid progenitor sejtek rosszindulatú átalakulása és ellenőrizetlen proliferációja blastsejtek megjelenését okozza a keringő vérben, amelyek a normál csontvelőt rosszindulatú sejtekkel helyettesítik.

ICD-10 kód

Az akut myeloblastos leukémia tünetei és diagnózisa

A tünetek közé tartozik a fáradtság, sápadtság, láz, fertőzés, vérzés és könnyű szubkután vérzések; a leukémiás infiltráció tünetei csak a betegek 5%-ánál jelentkeznek (gyakran bőrmegnyilvánulások formájában). A diagnózis felállításához meg kell vizsgálni a perifériás vér és a csontvelő kenetét. A kezelés magában foglalja az indukciós kemoterápiát a remisszió elérése érdekében és a remisszió utáni terápiát (őssejt-transzplantációval vagy anélkül) a visszaesés megelőzésére.

Az akut mieloid leukémia előfordulási gyakorisága az életkorral növekszik, és ez a leggyakoribb leukémia felnőtteknél, a betegség kialakulásának átlagos életkora 50 év. Az akut myeloid leukémia másodlagos rákként alakulhat ki kemoterápia vagy sugárterápia után különböző típusú rák esetén.

Az akut mieloid leukémia számos altípust foglal magában, amelyek morfológiájukban, immunfenotípusában és citokémiájában különböznek egymástól. A domináns sejttípus alapján az akut mieloid leukémiának 5 osztályát írták le: mieloid, mieloid monocitás, monocitás, eritroid és megakariocita leukémia.

Az akut promyelocyta leukémia különösen fontos altípus, és az összes akut myeloid leukémia %-át teszi ki. A betegek legfiatalabb csoportjában (átlagéletkor 31 év) és túlnyomórészt egy meghatározott etnikai csoportban (hispánok) fordul elő. Ez az opció gyakran véralvadási zavarokkal debütál.

Kihez forduljunk?

Akut myeloblastos leukémia kezelése

Az akut myeloid leukémia kezdeti terápiájának célja a remisszió elérése, és az akut limfoblasztos leukémiával ellentétben az akut mieloid leukémia kevesebb gyógyszerrel reagál. Az alapvető remisszió indukciós séma magában foglalja a citarabin vagy nagy dózisú citarabin folyamatos intravénás infúzióját 5-7 napon keresztül; Ez idő alatt a daunorubicint vagy az idarubicint intravénásan adják be 3 napig. Egyes kezelési rendek közé tartozik a 6-tioguanin, az etopozid, a vinkrisztin és a prednizolon, de ezeknek a kezelési rendeknek a hatékonysága nem tisztázott. A kezelés általában súlyos mieloszuppressziót, fertőző szövődményeket és vérzést eredményez; Általában sok időbe telik, amíg a csontvelő helyreáll. Ebben az időszakban elengedhetetlen a gondos megelőző és támogató gondozás.

Akut promyelocitás leukémia (APL) és az akut myeloid leukémia néhány más változata esetén a diagnózis idején disszeminált intravaszkuláris koaguláció (DIC) fordulhat elő, amelyet súlyosbít a leukémiás sejtek prokoagulánsok felszabadulása. Transzlokációval járó akut promyelocyta leukémiában (15; 17) az AT-RA (transzretinoinsav) alkalmazása elősegíti a blastsejtek differenciálódását és a disszeminált intravaszkuláris koaguláció korrekcióját 2-5 napon belül; daunorubicinnel vagy idarubicinnel kombinálva ez a kezelési séma a hosszú távú túlélő betegek 10%-ánál képes remissziót indukálni. Az arzén-trioxid hatásos az akut promielocitás leukémiában is.

A remisszió elérése után ezekkel vagy más gyógyszerekkel intenzifikációs fázist hajtanak végre; A nagy dózisú citarabin kezelések meghosszabbíthatják a remisszió időtartamát, különösen a 60 év alatti betegeknél. A központi idegrendszer károsodásának megelőzése általában nem történik meg, mivel a központi idegrendszer károsodása ritka szövődmény megfelelő szisztémás terápia mellett. Intenzíven kezelt betegeknél a fenntartó terápia semmilyen előnyt nem mutatott, de más helyzetekben hasznos lehet. Az extramedulláris elváltozások izolált recidívaként ritkák.

Az akut myeloblastos leukémia prognózisa

A remisszió indukciós aránya 50 és 85% között mozog. A hosszú távú betegségmentes túlélést az összes beteg %-ánál és a fiatal betegek %-ánál érik el, akiknek a kezelésében őssejt-transzplantáció is szerepelt.

A prognosztikai tényezők segítenek meghatározni a kezelési protokollt és annak intenzitását; Az egyértelműen kedvezőtlen prognosztikai tényezőkkel rendelkező betegek általában intenzívebb kezelésben részesülnek, mivel az ilyen kezelés lehetséges előnyei feltehetően indokolják a protokoll magasabb toxicitását. A legfontosabb prognosztikai tényező a leukémiás sejtek kariotípusa; a kedvezőtlen kariotípusok a t (15; 17), t (8; 21), inv16 (p13; q22). További kedvezőtlen prognosztikai tényezők az idősebb kor, a myelodysplasiás fázis, a másodlagos leukémia, a magas leukocitózis, az Auer-rudak hiánya. A FAB vagy a WHO besorolása önmagában nem jelzi előre a kezelésre adott választ.

Orvosszakértő szerkesztő

Portnov Alekszej Alekszandrovics

Oktatás: A Kijevi Nemzeti Orvostudományi Egyetem nevét viseli. A.A. Bogomolets, specialitás - „Általános orvostudomány”

LEUKÉMIA

M2 - akut sejtdifferenciálódással M3 - promyeloblastos leukémia, amelyet abnormális promyelociták jelenléte jellemez óriási granulátumokkal; gyakran kombinálják a granulátum tromboplasztikus hatása által okozott DIC-vel, ami megkérdőjelezi a heparin terápia célszerűségét. Az M: prognózisa kedvezőbb, mint az M0-M esetében A myelomonoblastos és monoblasztos leukémiát (M4 és M5) a nem eritroid sejtek, például a monoblasztok túlsúlya jellemzi. M< и М5 составляют 5-10% всех случаев острых миелобластных лейкозов. Частый признак - образование внекостномозговых очагов кроветворения в печени, селезёнке, дёснах и коже, гиперлейкоцитоз, превышающийх109/л. Чувствительность к терапии и выживаемость ниже, чем при других вариантах острых миелобластных лейкозов Эрит-ролейкоз (Мв). Вариант острого миелобластного лейкоза, сопровождающийся усиленной пролиферацией эритроидных предшественников; характерно наличие аномальных бластных ядросодержащих эритроцитов. Эффективность лечения эритролейкоза сходна с результатами терапии других подтипов или несколько ниже Мегакариобластный лейкоз (М7) - редкий вариант, сочетающийся с фиброзом костного мозга (острый миелосклероз). Плохо поддаётся терапии. Прогноз неблагоприятный.

C92 Mieloid leukémia C93 Myelocytás leukémia

C94 Egyéb meghatározott sejttípusú leukémia

C95 Meghatározatlan sejttípusú leukémia

A leukémia osztályozása az ICD-10-ben

R C91 limfoid leukémia [limfocita leukémia]

S C91.0 Akut limfoblaszt leukémia

S C91.1 Krónikus limfocitás leukémia

S C91.2 Szubakut limfocitás leukémia

S C91.3 Prolymphocytás leukémia

S C91.4 Szőrös sejtes leukémia

S C91.5 Felnőttkori T-sejtes leukémia

S C91.7 Egyéb meghatározott limfoid leukémia

S C91.9 Lymphoid leukémia, nem meghatározott

R C92 mieloid leukémia [mieloid leukémia]

S C92.0 Akut mieloid leukémia

S C92.1 Krónikus mieloid leukémia

S C92.2 Szubakut myeloid leukémia

S C92.3 Mieloid szarkóma

S C92.4 Akut promielocitás leukémia

S C92.5 Akut myelomonocytás leukémia

S C92.7 Egyéb mieloid leukémia

S C92.9 Myeloid leukémia, nem meghatározott

R C93 Monocitás leukémia

S C93.0 Akut monocitás leukémia

S C93.1 Krónikus monocitás leukémia

S C93.2 Szubakut monocitás leukémia

S C93.7 Egyéb monocitás leukémia

S C93.9 Monocitás leukémia, nem meghatározott

R C94 Egyéb meghatározott sejttípusú leukémia

S C94.0 Akut eritremia és eritroleukémia

S C94.1 Krónikus eritremia

S C94.2 Akut megakarioblasztos leukémia

S C94.3 Hízósejtes leukémia

S C94.4 Akut panmyelosis

S C94.5 Akut myelofibrosis

S C94.7 Egyéb meghatározott leukémia

R C95 Meghatározatlan sejttípusú leukémia

S C95.0 Meghatározatlan sejttípusú akut leukémia

S C95.1 Nem meghatározott sejttípusú krónikus leukémia

S C95.2 Nem meghatározott sejttípusú szubakut leukémia

S C95.7 Egyéb, nem meghatározott sejttípusú leukémia

S C95.9 Leukémia, nem meghatározott

A krónikus myeloid leukémia (CML) tumor jellegű, klonális jellegű betegség, amely a mielopoézis korai prekurzoraiból ered, melynek morfológiai szubsztrátja túlnyomórészt érő és érett granulociták.

A mai napig nem tanulmányozták részletesen. A betegség kialakulásában nagy jelentősége van:

· kémiai tényezők hatása, amelyek növelik a kromoszóma-rendellenességek számát.

Az emberek gyakrabban elakadnak. Férfiaknál és nőknél egyaránt gyakori. Az összes hemoblasztózis között az 5. helyen áll. Évente lakosságonként 1-1,5 esetet regisztrálnak.

CML-ben szenvedő betegeknél egy specifikus kromoszóma-rendellenességet fedeztek fel a vérképző őssejtekben - a Philadelphia kromoszómában (22q-, Ph'). A t(9;22)(q34;qll) reciprok transzlokációhoz kapcsolódik, ami egy fúziós gén kialakulásához vezet. BCR-ABL b3a2 és/vagy b2a2 típus, amely döntő genetikai eseménynek bizonyult a CML kialakulásában, és kulcsfontosságú patogenetikai szerepet játszik a betegség klinikai megnyilvánulásainak későbbi kialakulásában.

A fúziós gén terméke BCR-ABL a citoplazmatikus fúziós onkoprotein p210 BCR - ABL, más hibrid onkoproteinek ritkábban képződnek (p230 BCR - ABL, p190 BCR - ABL). Ez az onkoprotein túlzott tirozin-kináz aktivitással rendelkezik, és a CML szinte minden fő klinikai megnyilvánulásáért felelős.

A BCR-ABL fehérje ellenőrizetlen autonóm hatást fejt ki a CML-ben aktivált proto-onkogén közösségben a fő sejtfunkciókra MYС, CRKL, GRB2, KIT, VAVÉs MYB t ami a mieloid sejtek ellenőrizetlen proliferációjához vezet a fő jelátviteli útvonalon - a mitogén-aktív protein kinázok MAPK aktiválásán keresztül. A neoplasztikus mielociták stromasejtekhez való tapadásának megszakadása és az apoptózis folyamatai is megszakadnak.

· Klonális jellegű tumor progresszió. A kezdeti stádiumban - monoklonális daganat, a terminális időszakban - poliklonális, szarkómás sejtnövekedés fordulhat elő.

· A daganatsejtek számának 1 μl feletti növekedése szervi véráramlási zavarokhoz, elsősorban agyi véráramlási zavarokhoz vezethet.

· Magas leukocitózis és sejtlebomlás esetén a húgysav növekedése és vesekőképződés lehetséges.

· DIC szindróma kialakulása.

· Hiperplasztikus szindróma különböző szervek és szövetek mieloid infiltrációjával (periosteum, ízületek, neuroleukémia).

Jelenleg a fejlett, átmeneti és terminális szakaszokat különböztetik meg.

1. szakasz, bővítve. Az előrehaladott stádium kezdeti szakaszában a betegek jóléte nem zavart. Nincsenek klinikai tünetek. A megelőző vizsgálat vagy bármely betegség kezelése során végzett laboratóriumi vizsgálat során véletlenül leukocitózist észlelnek. Általában 1 µl tartományban. Jellemzője a leukocita képlet mielocitákra és promyelocitákra való eltolódása, a leukocita/eritrocita arány növekedése a csontvelőben. A „Philadelphia kromoszóma” a granulocitákban és a csontvelő sejtekben található. Ennek a szakasznak az időtartama körülbelül 4 év.

2. szakasz, átmeneti. Az éretlen formák megnövekedett tartalma (a promielociták aránya akár 20-30%), bazofília. Blast sejtek a csontvelőben akár 10%.

A legkorábbi klinikai tünetek: gyengeség, fáradtság, izzadás; néha korai tünet lehet a lép megnagyobbodása miatti tompa fájdalom vagy elnehezülés a bal hypochondriumban.

A betegség klinikai képében a következő szindrómákat lehet megkülönböztetni:

1) mérgezés (izzadás, gyengeség, láz a fertőzés nyilvánvaló jelei nélkül, fogyás);

2) disszeminált véralvadás által okozott hemorrhagiás szindróma;

3) fertőző szindróma (torokfájás, hörghurut, tüdőgyulladás, egyéb fertőző betegségek, szepszis);

4) húgysav-diatézis szindróma, amely a tumorsejtek nagymértékű pusztulásával jár,

5) hiperpláziás szindróma (megnagyobbodott lép, máj, ritkán a betegség kezdetén és jellemzőbb a terminális időszakban - megnagyobbodott nyirokcsomók, bőrleukémia, periosteum, idegszövet beszűrődése).

1. Neutrophil leukocytosis balra tolódással mielocitákra és promyelocitákra.

2. A vörös vér nem változik a betegség kezdetén.

3. A vérlemezkék kezdetben változatlanok vagy mérsékelten csökkentek.

A granulociták szinte teljesen helyettesítik a zsírszövetet. A leuko/eritro csírák aránya 10:1-20:1 (általában 3-4:1).

Máj és lép

Jellegzetes myeloid infiltráció.

A kóros folyamat fokozatosan előrehalad, a gyógyszeres kezelésre való érzékenység csökken. Növekszik a vérszegénység és a thrombocytopenia, a mérgezés.

1 - Ph kromoszóma nélkül (Philadelphia kromoszóma). A betegek kedvezőtlen lefolyása és rövid várható élettartama jellemzi. A hepato- és splenomegalia korán jelentkezik. A várható élettartam gyermekeknél 5-6 hónap, felnőtteknél - 1,5-2 év.

2 - Ph + kromoszómával, időseknél gyakrabban a betegség lefolyása lassú. Ha azonban a Ph kromoszóma a vérlemezkék számának csökkenésével párosul, a prognózis kedvezőtlen.

Philadelphia kromoszóma - 22-es kromoszómapár, amelynek lerövidített hosszú karja van - a 9-es kromoszómáról a 22-re, illetve a 22-es kromoszómáról a 9-es részeire történő transzlokáció eredménye. Ennek eredményeként egy hibrid „kiméra” gén képződik, melynek neve bcr/abl. A patológiás p210 fehérje szintézisét kódolja, amely egy megnövekedett aktivitású tirozin kináz, amely az ATP tirozinba való átviteléért felelős különböző intracelluláris fehérjéken. A foszforiláció folyamatában számos fehérje aktiválódik, és a sejt normális működése megszakad, ami a sejtek rosszindulatú átalakulásához vezet.

Az elmúlt évtizedekben a CML progresszív (akcelerált) szakaszát különböztették meg, amelyben a betegség lefolyása malignusabbá válik. Ebben a tekintetben az orvosi taktika gyökeres megváltoztatására van szükség.

A gyorsulási fázis legfontosabb jele a blastsejtek és promyelociták számának növekedése a perifériás vérben és/vagy a BM-ben. Véleményünk szerint a progresszív (akcelerált) fázist jelzi, ha a perifériás vérben és/vagy a BM-ben 15% vagy több ilyen sejt (vagyis a blasztsejtek és promyelociták összszáma) kimutatható. Ezenkívül a leukociták számának terápiarezisztens növekedése, fokozódó thrombocytosis vagy thrombocytopenia, vérszegénység jelentkezik, amely nem kapcsolódik a terápiához.

Egy előre nem látható szakaszban a monoklonális daganat poliklonálissá válik. Ez jellemzi a betegség kialakulásának következő szakaszát - a terminális időszakot. A terminális időszakot a következők jellemzik:

1. A lép gyors növekedése.

2. Hőmérséklet emelkedés.

3. Csontfájdalom.

4. Blast krízisek (5%-nál több blastos sejtek megjelenése a vérben).

5. Szarkómás növekedési gócok.

6. Leukémiák előfordulása a bőrben.

8. Tűzálló a mielozánra.

9. Metaplasztikus vérszegénység (Hb<110 г/л) и тромбоцитопении (менее 100*10 9 /л)

Átfogó vizsgálat alapján állapítják meg: tipikus klinikai kép, vérvizsgálat, csontvelői változások, esetenként a Ph + kromoszóma meghatározása. Néha meg kell különböztetni az osteomyelofibrosistól (a trepanobiopszia során csontvelő-fibrózist észlelnek).

A diagnózis kritériumai a következők:

1. 1 µl-nél nagyobb leukocitózis.

2. Megjelenés a fiatal formák vérében: myeloblastok, promyelociták, mielociták, metamyelociták.

3. A csontvelő mieloid proliferációja.

4. Ph + -kromoszóma jelenléte.

5. Megnagyobbodott lép és/vagy máj.

Gyakran szükség van differenciáldiagnózisra az IMF és a CML között. A fő különbségi jellemzőket a táblázat tartalmazza.

Az idiopátiás myelofibrosis és a krónikus mieloid leukémia fő klinikai és laboratóriumi tünetei

1. szakasz. Enyhe leukocitózis esetén, különösen időseknél: helyreállító terápia, vitaminok, adaptogének.

40-50*10 9 /l leukocitózis esetén 1 mg/ttkg hidroxi-karbamidot vagy 4 mg/nap biszulfánt alkalmazunk orálisan. Az adagokat úgy választják meg, hogy a leukociták szintje körülbelül 20 * 10 9 / l legyen.

2. szakasz. Választható gyógyszerek:

Hidroxi-karbamid 1 mg/nap dózisban (általában napi 1 mg fenntartó adag).

· Α-interferon. Adagolás 5-9 millió egység heti 3 alkalommal IM. Lehetővé teszi a hematológiai remisszió elérését a betegek százalékánál.

Ha a lép jelentősen megnagyobbodott, sugárterápia lehetséges.

3. szakasz. Az akut leukémia kezelésében használt gyógyszereket alkalmazzák.

A myelosan továbbra is őrzi pozícióját a CML-ben szenvedő betegek kezelésében. Alkalmazása olyan betegeknél indokolt, akik súlyos mellékhatások vagy egyéb okok miatt nem kezelhetők interferon-α-val vagy hidroxi-karbamiddal.

Leukocitózisra írták fel több mint ezer. 1 µl.mg-ban naponta.

Leukocitózissal. 1 μl-ben - az adagot napi 6 mg-ra emelik.

Nagyobb leukocitózis esetén - akár 8 mg naponta.

Általában, amikor a leukociták száma csökken (4-6 hét), hetente egyszer mg fenntartó adagot írnak elő. A leukociták szintjét ezren belül tartják. 1 µl-ben. Nem szabad megfeledkezni arról, hogy a gyógyszer adagolása az eltérő egyéni érzékenység miatt eltérő lehet.

Ha a mielozán nem elég hatékony, a következőket írják elő:

Myelobromol adag mg / nap. 2-3 hét elteltével a fenntartó terápiát 5-10 naponta egyszer ugyanabban az adagban adják.

Dopan - jelentős splenomegalia esetén, ha más gyógyszerek hatástalanok. 6-10 mg naponta 1 alkalommal 4-10 naponként.

A kezelést leállítják, ha a leukociták száma 5-7 ezerre csökken 1 μl-ben. Fenntartó terápia 6-10 mg 2-4 hetente egyszer.

A hexafoszfamid (hidroxi-karbamid) a választott gyógyszer. Leukocitózis esetén több mint 1 μl - 20 mg naponta; 1 μlmg-ban hetente kétszer; Ha a leukocitaszint 1 µl, a gyógyszert abba kell hagyni. Fenntartó terápia mg 5-15 naponta.

Citozin arabinád és itron A CML-ben szenvedő betegek kezelésében

A citozin arabinád szelektíven gátolja a transzformált Ph+ progenitor sejtek proliferációját.

Α-interferon (itron A). Kifejezett antiproliferatív aktivitással rendelkezik. A gyógyszer nagyon hatékony a CML-ben szenvedő betegek kezelésében. Kimutatták, hogy monoterápiával egy hónappal meghosszabbíthatja a betegek életét, és késlelteti a blastos krízis kialakulását. A teljes citogenetikai válaszreakciót mutató betegeknél a legnagyobb az élettartam növekedés, a 10 éves túlélési arány %.

Gleevec. Új irány a CML-es betegek kezelésében a p210 bcr/abl fehérje aktív helyének megfelelő gyógyszerek alkalmazása (imatinib mesylate, Gleevec). Az STI 571 molekula (egy 2-fenil-amino-piridin származék) beépül a mutáns abl-tirozin kináz molekulába, blokkolva a tirozin foszforilációját. Ezeknek a gyógyszereknek az alkalmazása blokkolja az intracelluláris fehérjék foszforilációs folyamatait, ami a sejtek pusztulását okozza, főként a patológiás bcr/abl fehérjét tartalmazó sejtek. Ezeknek a gyógyszereknek a nagy hatékonysága a CML minden szakaszában bizonyított. A gyógyszert 400 mg/m2 dózisban írják fel 28 napig. Robbanásválság esetén a dózis 600 mg/m2 lehet.

Kezelés a terminális időszakban

Alacsony dózisú citozin-arabinozid interferon-α-val szintén alkalmazható a progresszív fázisban (a megközelítések megváltoztatását a CML progressziójának első jeleinél kell elkezdeni).

Ha ez a megközelítés nem hatékony, polikemoterápia alkalmazható. A leggyakrabban használt antraciklin antibiotikumok és citozin-arabinozid hagyományos kombinációi, például az „5+2”. Ez a program 60 mg/m2 rubomicint vagy az antraciklin csoportba tartozó más gyógyszert tartalmaz megfelelő dózisban az első két napon és 100 mg/m2 citozin-arabinozidot naponta kétszer öt napon keresztül. Ha ez a kezelési rend nem elég hatékony, akkor a „7+3” kombináció alkalmazható.

Ha CML blasztos krízis lép fel (a blasztok és/vagy promyelociták száma a csontvelőben és/vagy a perifériás vérben meghaladja a 30%-ot), a blastos krízis immuncitokémiai változatának meghatározása után terápiás taktikát dolgoznak ki. A jelenlegi álláspont továbbra is az, hogy a CML blast krízis kezelése az akut leukémia kezelésében alkalmazott programok szerint történik.

Leukocitoferézis. Nagyszámú leukociták és vérlemezkék esetén végezzük, különösen az agyi véráramlás meglévő rendellenességei esetén (fejfájás, halláskárosodás stb.).

Az extramedulláris daganatképződmények (mandula hiperplázia, neuroleukémia, csontfájdalom) kezelése sugárterápiával is elvégezhető.

Splenectomiát léprepedés, súlyos hasi diszkomfort, ismételt periszplenitisz esetén végezzük; hipersplenizmus jelenségei.

Csontvelő-transzplantáció Az allogén hematopoietikus sejtek transzplantációja sokáig az egyetlen módszer maradt a CML-ben szenvedő betegek gyógyítására. Ennek a műtétnek az a lényege, hogy a HLA (humán leukocita antigének) rendszer szerint kompatibilis donort választanak ki a páciens számára. BM-et gyűjtenek a donortól, vagy perifériás őssejteket izolálnak. A páciens aszeptikus dobozban kondicionáláson (előkészítésen) esik át, amely citosztatikus gyógyszerek szubletális dózisait tartalmazza, néha sugárzással kombinálva. A kondicionálás célja a leukémiás sejtek kóros klónjának kiirtása (megsemmisítése). Ezt követően transzplantációt hajtanak végre, amely külsőleg donorvér (allogén transzplantáció esetén) intravénás infúziónak tűnik.

Sajnos előfordulhat, hogy ennek a módszernek az alkalmazása nem minden betegnél hatékony.

Új irányok a CML-es betegek kezelésében

Számos új gyógyszer alkalmazásáról tárgyalnak jelenleg is: citosztatikus szerek, jelátviteli gátlók (kivéve Gleevec), farnezil-transzferáz vagy geranilgeranil-transzferáz inhibitorok, beleértve a BCR-ABL tirozin-kináz, JAK2 tirozin-kináz és scr-kináz új inhibitorait, amelyek bcr-abl lebontás fokozása, proteázgátlók, immunkezelések.

Az átlagos várható élettartam kemoterápiával 3-4 év. Az első robbanásválság után a várható élettartam általában körülbelül 12 hónap. A halál okai: fertőző és vérzéses szövődmények a terminális időszakban.

A terápiás taktika meghatározásakor figyelembe veszik a kockázati csoportokat: a magas kockázat az allogén BM vagy perifériás őssejtek korai transzplantációjának szükségességét, valamint az aktívabb terápia szükségességét jelzi.

A rossz prognózis legspecifikusabb jelei a következők:

  • 60 éves és idősebb korosztály.
  • A blasztózis a perifériás vérben 3% vagy több, vagy a BM 5% vagy több.
  • Basofilek a perifériás vérben 7% vagy több, vagy a testtömegben 3% vagy több.
  • Trombocitózis 700*10 9 /l vagy több.
  • Splenomegalia - a lép 10 cm-rel vagy annál jobban kinyúlik a bordaív széle alól.

A krónikus limfocitás leukémia (CLL) CD5+ pozitív B-sejtek daganata.

A krónikus limfocitikus leukémia (CLL) - a WHO besorolása szerint "krónikus limfocitás leukémia/kis limfocita limfóma" - a limfoid szövet betegsége, amelyet klonális proliferáció és a hosszú életű daganatos limfociták folyamatos felhalmozódása jellemez a perifériás vérben, a csontvelőben (BM). ), nyirokcsomókban, lépben, májban, majd más szervekben és szövetekben.

Az előfordulás népességenként 0,08-2,2. Ez a leukémia leggyakoribb típusa Európában és Észak-Amerikában. Az összes leukémia 30%-át teszi ki.

Átlagos életkor. Etiológiája nincs meghatározva.

Jelenleg a CLL természetét a legpontosabban azok a biológiai fogalmak tükrözik, amelyek az apoptózis mechanizmusaira, a B-sejtek sejtciklusára, a tumor B-sejtek genetikai különbségeire és a B-sejtek genetikai különbségeire, valamint kromoszóma-rendellenességek, CD38, ZAP -70 és más jelátviteli molekulák túlzott expressziója, valamint adatok a B-sejtek funkcionális aktivitásának folyamataiban és mikrokörnyezetükben a nyirokcsomókban és a BM-ben.

A limfociták különböző klónjainak tumornövekedése Különböző esetekben a limfociták különböző klónjai vesznek részt a daganatos folyamatban. Szigorúan véve a „krónikus limfocitás leukémiának” számos betegségből kell állnia, bár számos közös vonásuk van.

A patogenezis fő elemei a T - vagy B - limfocita klónok hiperpláziája, kifejezett leukocitózissal és limfocita infiltrációval a csontvelőben, nyirokcsomókban, lépben és májban.

Hematopoiesis depressziója.Számos ok miatt: az immunmechanizmus, melynek eredményeként antitestek képződnek a csontvelő vérképző sejtjei vagy érett vérelemek ellen (a hemolízis autoimmun jellegét pozitív direkt Coombs-teszt igazolja); a leukémiás sejtek citolitikus hatása, ha ölő tulajdonságokkal rendelkeznek; a T-szuppresszor sejtek (nem tumoros természetű) hatása, amely a sejtek, az eritropoézis prekurzorai proliferációjának elnyomásához vezet; hipersplenizmus; a normál hematopoiesis kiszorítása tumorsejtek által .

Leukémiás sejtek beszivárgása az idegtörzsekbe és a központi idegrendszerbe.

DIC szindróma kialakulása.

Különböző szervek összenyomása nyirokcsomók által (különösen a mediastinum).

Klinikai kép (tipikus)

A leukociták számának akár ezres növekedése hosszú évekig is fennállhat. 1 µl-ben, melynek 60-80%-a limfocita. A betegséget gyakran a megelőző vizsgálatok során észlelik.

A leukocitózis torokfájással és fertőző betegségekkel növekszik, és gyógyulás után csökken.

A nyirokcsomók fokozatosan megnagyobbodnak, különösen a nyak és a hónalj területén, majd a folyamat átterjed a mediastinumba, a hasüregbe és az ágyék területére.

Ezenkívül előfordulnak a leukémiára jellemző nem specifikus jelenségek: fokozott fáradtság; gyengeség; izzadó

A betegség korai szakaszában nincs vérszegénység vagy thrombocytopenia. Néha még 100 ezer leukocita esetén sincs vérszegénység.

Csontvelő-punkció (BM) - a limfociták számának növekedése a mielogramban több mint 30%.

A BM trefin biopsziája a limfoid sejtek jellegzetes proliferációja, gyakran diffúz.

Vérvizsgálat - a limfociták számának növekedése. Ezenkívül vannak a limfociták roncsolt magjai - Gumprecht árnyékai (ez egy műtermék, vérkenet készítésekor keletkeznek a limfociták megnövekedett elpusztíthatósága miatt). A betegség előrehaladtával egyes prolimfociták és limfoblasztok kezdenek megjelenni a vérben.

Gyakran megfigyelhető a retikulociták számának növekedése. Az 1. év során az esetek 60%-ában nem érintett a vörösvér. 3-7 éves betegséggel a vérszegénységben szenvedők száma 70%-ra nő.

A thrombocytopenia kialakulása elsősorban a leukémiás folyamat előrehaladásának felel meg.

1. Kezdeti szakasz.

A). Több nyirokcsomó, egy vagy több csoport enyhe megnagyobbodása.

b). Leukocitózis ezren belül. 1 mikronban.

V). A leukocitózis nem növekszik több hónapon keresztül.

G). A beteg szomatikusan kompenzált.

2. Kiterjesztett szakasz.

A). Növekvő leukocitózis.

b). A nyirokcsomók progresszív megnagyobbodása.

V). Ismétlődő fertőzések megjelenése.

G). Autoimmun citopéniák.

3. Terminál fokozat.

A terminális szakasz fő kritériuma a CLL rosszindulatú átalakulása. A morfológiai kép a normál hematopoietikus csírák gátlása és a csontvelő lokális helyettesítése blast sejtekkel. A CLL-nek a terminális stádiumba való átmenetét gyakran a nyirokcsomók szarkómás növekedése vagy ritkábban blastos krízis kíséri.

0. stádium, amelyben csak /L-nél nagyobb limfocitózis van a vérben és több mint 40% a csontvelőben, a betegek átlagos túlélése a betegség ezen szakaszában megegyezik a populációéval.

I. szakasz - limfocitózis és nyirokcsomó-megnagyobbodás jellemzi, átlagos túlélése 9 év.

II. stádium – limfocitózissal, lép- és/vagy hepatomegáliával, függetlenül a nyirokcsomók megnagyobbodásától és a medián túléléstől 6 év.

III. stádium – limfocitózissal és a hemoglobinszint 11 g/dl alá csökkenésével.

IV. stádium – limfocitózissal és a thrombocytaszám 100*10 9 /l alá csökkenésével, függetlenül a nyirokcsomók és szervek megnagyobbodásától, és a medián túlélés mindössze 1,5 év.

1. Hypogammaglobulinemia. Csökkent immunglobulin tartalom. Fokozott érzékenység a fertőzésekre (tüdőgyulladás, mandulagyulladás, pyelonephritis és egyéb fertőzések). Súlyos, néha végzetes szövődmény a herpes zoster.

2. Schonlein-Henoch szindróma.

4. A VIII pár agyideg beszűrődése halláskárosodással.

5. A neuroleukémia kialakulása. A klinikai kép nem különbözik az akut leukémiától.

6. Mellhártyagyulladás (para - vagy metapneumoniás banális fertőzéssel; tuberkulózisos mellhártyagyulladás).

7. Kimerültség, hipoalbuminémia.

8. Infiltráció miatt kialakuló krónikus veseelégtelenség. Klinika: hirtelen anuria.

Neoplazmák szarkómás növekedése (nyirokcsomók, lép stb.).

A CLL megkülönböztető jellemzője a perifériás vér leukociták számának növekedése, jelentős számú kis érett limfocitával - több mint 5 * 10 9 / l (akár 95%), Gumprecht „árnyékainak” azonosítása (megsemmisült az előkészítés során limfociták kenetéből) és a limfoid sejtek jellegzetes immunfenotípusának jelenléte - CD19, CD20, CD23 és CD5. A B-CLL betegek 7-20%-ánál hiányzik a CD5 (amelynek jelenléte autoimmun reakciókkal jár).

1. Abszolút limfocitózis a vérben (több mint 10*10 9 /l).

2. A csontvelő-aspirátumban a limfociták száma több mint 30%.

3. A nyirokcsomók és a lép megnagyobbodása opcionális tünet, de ha van, akkor limfociták proliferációját mutatják ki bennük.

4. Gumprecht-féle árnyékok vérkenetekben (segédjel).

5. Leukémiás sejtek B-sejt klónjának immunológiai megerősítése, esetenként monoklonális immunglobulinok szekréciójával.

2. Progresszív (klasszikus).

6. Citolízissel szövődött krónikus limfocitás leukémia.

8. Paraproteinémiával járó CLL.

9. Szőrös sejtes leukémia.

10. T-sejt forma.

A CLL különböző formáinak lefolyásának jellemzői

1. Jóindulatú forma:

Nagyon lassú áramlás;

A nyirokcsomók enyhén megnagyobbodtak;

A limfociták lassú növekedése.

2. Progresszív forma (klasszikus):

A kezdet ugyanaz, mint a klasszikus formánál;

A limfociták számának növekedése hónapról hónapra;

Megnagyobbodott nyirokcsomók.

3. Daganat forma:

A nyirokcsomók jelentős megnagyobbodása;

Megnagyobbodott lép (jelentős vagy mérsékelt);

A mérgezés hosszú ideig rosszul fejeződik ki.

4. Splenomegaliás forma:

A nyirokcsomók mérsékelt megnagyobbodása;

A lép jelentős megnagyobbodása.

(A lép limfocitómától való megkülönböztetésére - csontvelő-trefinnel, nyirokcsomó-biopsziával - a nyirokelemek diffúz burjánzása figyelhető meg).

5. A CLL csontvelői formája:

Gyorsan progresszív pancitopénia;

a csontvelő (teljes vagy részleges) cseréje érett limfocitákra;

A nyirokcsomók és a lép nem növekszik meg.

6. Citolízissel komplikált CLL:

Hemolízis és vérszegénység (emelkedett bilirubin, retikulocitózis) jellemzi;

Közvetlen Coombs-teszt az immunformára;

Thrombocytopenia (magas vagy normál megakariocita-tartalommal a csontvelőben, jobban kimutatható trepanátban).

7. Prolimfocita forma:

A prolimfociták dominálnak (a vérkenetben a tumorsejtekben nagy, tiszta sejtmag található);

A perifériás nyirokcsomók mérsékelt megnagyobbodása;

Az immunglobulinok (általában IgM) monoklonális hiperprodukciója.

8. Paraproteinémiával járó CLL:

A CLL szokásos klinikai képe;

Monoklonális M - vagy G - gammopathia (az első esetben - Waldenström-kór);

Megnövekedett vér viszkozitása.

9. Szőrös sejtforma:

Sejtmorfológia: homogén sejtmag, blastokra és széles csipkés citoplazmára emlékeztető, töredezett, bolyhokra és szőrszálakra emlékeztető csírákkal. Jellemző a savas foszfatázzal szembeni fényes diffúz reakció;

A nyirokcsomók normál mérete;

A tanfolyam változó (néha évekig nincs előrehaladás).

A bőrszövet mély rétegeinek beszivárgása;

Vérkép: leukocitózis, neutropenia, vérszegénység.

A CLL kezelésének általános elvei

A betegség korai stádiumában, enyhe, 20-30*10 9 /l tartományba eső leukocitózis esetén citosztatikus terápia nem történik A CLL citosztatikus terápia megkezdésének indikációi:

1) általános tünetek jelenléte: fáradtság, izzadás, fogyás;

2) vérszegénység vagy thrombocytopenia, amelyet a csontvelő leukémiás sejtek infiltrációja okoz;

3) autoimmun vérszegénység vagy thrombocytopenia;

4) masszív lymphadenopathia vagy splenomegalia, ami kompressziós problémákat okoz;

5) nagyszámú limfocita a vérben (több mint 150*10 9 /l);

6) a limfociták abszolút számának megduplázódása a vérben kevesebb, mint 12 hónap alatt;

7) fokozott fogékonyság a bakteriális fertőzésekre;

8) a csontvelő masszív limfocita beszűrődése (a limfociták több mint 80%-a a mielogramon);

9) komplex kromoszóma-rendellenességek jelenléte;

10) a betegség előrehaladott stádiuma: Rai szerint III–IV.

Chlorbutin (chlorambucil, leukeran) 0,1-0,2 mg/ttkg naponta megnagyobbodott nyirokcsomók és lép esetén.

Ciklofoszfamid - 2 mg / kg / nap. Leukeránnal szemben rezisztens CLL-rel, valamint a leukocitózis növekedésével, a nyirokcsomók vagy a lép jelentős megnagyobbodásával.

Szteroid hormonok - a nyirokcsomók gyors megnagyobbodása, a mérgezés eltávolítása, a jólét javítása, a hőmérséklet normalizálása. Az ebbe a sorozatba tartozó gyógyszerekkel végzett terápia azonban nagyon veszélyes a lehetséges szövődmények miatt.

Fludarabin (Fludar), pentosztatin, kladribin A purin nukleozidok csoportjába tartoznak. A gyógyszereket adenozin helyett DNS-be és RNS-be építik be. Gátolja a DNS és RNS szintéziséhez szükséges számos enzim működését.

A fludarabinnal végzett kezelés jobb, mint az egyes gyógyszerek és a kemoterápia. Ezért még egy új, fludorabin korszakról beszélnek a CLL kezelésében. Egyszerre intravénásan vagy 30 percen keresztül csepegtetve 25 mg/m2-rel, 5 egymást követő napon, 28 naponként. Alopecia a betegek 2%-ánál alakul ki. A gyógyszer nefrotoxikus, és nem írható fel, ha a clearance 30 ml/perc. A leggyakoribb mellékhatás a mieloszuppresszió (Hb<6,5, лейкоциты< 1000 в 1 мкл, тромбоциты менее 25*10 9 /л).

A sugárterápiát a következő esetekben végezzük:

A nyirokcsomók jelentős megnagyobbodása, citopenia;

Vagy magas fehérvérsejtszám és thrombocytopenia esetén;

A lép jelentős mérete;

Leukemoid infiltráció az alsó törzsek területén.

Egyszeri adag 1,5-2 g. Totalgr. A csigolya megsemmisítésével 25 g-ig.

Splenectomia. A javallatok közé tartozhat súlyos splenomegalia és citopenia; - óriás lép, gyors növekedése, szívrohamok, tartós fájdalom.

Leukoferézist hajtanak végre, ha megnövekszik a leukociták száma és alacsony a gyógyszeres kezelés hatékonysága (gyakran hatékony thrombocytopenia és agranulocytosis esetén).

A plazmaferézist a JgM és JgG szekréciója által okozott megnövekedett viszkozitással végezzük; polyneuritis (gyakran immunkomplexek okozzák).

Csontvelő-transzplantáció

Akkor javasolt, ha a fludarabin-terápia hatástalan.

A legtöbb CLL-ben szenvedő beteg a diagnózis után 3-5 évig él. A betegség lassú progressziójával, amely időseknél kezdődik, a várható élettartam körülbelül 10 év.

A rossz prognózis jelei:

  • többszörös kromoszóma-rendellenességek,
  • a betegség gyors előrehaladása,
  • súlyos autoimmun reakciók,
  • fiatal kor.